Nueroscience: Questions Block 1, Block 2 (Class Questions)

Ace your homework & exams now with Quizwiz!

a 26-year-old woman with type 1 diabetes diagnosed at the age of 15, presented with burning pain in her lower extremities and upper extremity digital paresthesias that made her work as a dental hygienist difficult. Recently, her family had noted that she seemed to be stumbling at times. Physical examination reveals decreased pain sensation over both feet and bilateral absence of ankle reflexes. This patient's neurological symptoms are most likely associated with which of the following? A.Endoneural inflammatory infiltration B.Mutation of a myelin protein gene C.Endoneural arteriole hyalinization D.Nerve compression within an anatomic compartment E.Poor signal transmission at the neuromuscular junction

A.Endoneural inflammatory infiltration B.Mutation of a myelin protein gene C.Endoneural arteriole hyalinization (Diabetic Person) D.Nerve compression within an anatomic compartment E.Poor signal transmission at the neuromuscular junction

A 33-year-old man is noted to have exacerbations of weakness. On the examination Lhermitte 's sign is positive. Scattered, periventricular, plaque-like lesions found on this patient's MRI are most likely to demonstrate: A.Eosinophilic neuron degeneration B.Astrocyte shrinking C.Oligodendrocyte depletion D.Local hemorrhages

A.Eosinophilic neuron degeneration B.Astrocyte shrinking C.Oligodendrocyte depletion D.Local hemorrhages Multiple sclerosis

A 25-year-old man developed multiple neurological symptoms over the past year. During examination you bend down patients head and patient felt sudden, uncomfortable sensation that travels from his neck down to his spine- Lhermitte's sign. Which of the following is the most likely morphologic basis of this patient's condition? A.Neuron loss B.Axon disruption C.Axon denudation D.Astrocyte degeneration E.Nerve compression F.Synaptic transmission failure G.Muscle damage

A.Neuron loss B.Axon disruption C.Axon denudation D.Astrocyte degeneration E.Nerve compression F.Synaptic transmission failure G.Muscle damage

A 35 year old female came to GP for evaluation of her long-term neurologic complaints. She complains of heat intolerance precipitating a stumbling gait and a tendency to fall. Her visual acuity also seemed to change periodically during several years. Physical examination shows slow adduction of the left eye when the patient looks to the right. Which of the following is the most likely morphologic basis of this patient's condition? A.Neuron loss B.Axon disruption C.Axon denudation D.Astrocyte degeneration E.Nerve compression F.Synaptic transmission failure G.Muscle damage

A.Neuron loss B.Axon disruption C.Axon denudation D.Astrocyte degeneration E.Nerve compression F.Synaptic transmission failure G.Muscle damage Multiple Scelerosis patients do not like heat. Multiple Sceloris is Axon Denudation (demylenation) - There is no nerve compression.

Where is a Horner's lesion?

Above T1 Same side lesion - the is above. It is a lateral issue.(lateral syndrome) -

Information

All cranial never lesion and body are contralateral All lesion are on the same side Lesion is contalaterl if lesion is in the cortex because above Everythign is the below is the same like in the bod. Everythign above, is contralateral. Everyting below the lesion has the same side on the breian stem and body. Evetying on the level of the lesion, it is the same side. Fot the body, it si contralateral 7th and 12th crainal 7th crainla never has two part (nulue that innverates the upper part of the boy), the lower part is the face. If lesion in pon, half the body does no twork If the lesion is above in the cortivulncule pathway an damage to the mouth and contralateral side. Half of the face will not work but very damage If the lesion is the nerves, it is bell palsy. The tongue, if the lesion in the medulla the tongue deviate to the opposite side. The tongue is going to lesion. If the lesion is above the medulla, the tongue will more controlater. The tongue will localize in the middle in the medulla. Bell Palsy is damage to the 7th craninal never. It has problesm in th

What is Resting tremor/Poverty Movement: Parkinson SYndrome

An hallmark feature of Parkinson's disease is the resting tremor - involuntary, oscillating movements at rest, often of the hands or head and neck. It is not a resting tremor if it occurs during execution of a movement! Poverty Movement Limited or no movement - often described by the person as becoming "stuck" in position - also is characteristic of Parkinson's disease. Remember that the basal ganglia are responsible for initiating a change in movement patterns. Here, the individual is attempting to rise from a chair - a complex motor sequence where timing of the component movements is critical.

11 What is the location of termination for the Cuneocerebellar pathway?

Anterior lobe (cerebellum)

A 56-year-old man presents to the emergency room with complaints of difficulty walking and frequent falls over the past 48 hours. He has a 30-year history of chronic alcoholism. His nutritional status is poor and he admits to spending all of his money on alcohol instead of buying food to eat. Prior to his presentation, he describes an alcoholic binge and several days of anorexia. On physical examination he is noted to walk with a wide-based, irregular gait. He has poor coordination on tests requiring rapid leg movements. Based on the patient's history of severe alcoholism and malnutrition, alcoholic cerebellar degeneration was thought to be the most likely diagnosis. After treatment with vitamin B1 (thiamine), aggressive hydration with intravenous fluids, and good nutrition, the patient's symptoms gradually improved and he was discharged to alcohol treatment program. ◆ Which part of the cerebellum is most likely affected? ◆ If left untreated, what disease is he at risk for?

Anterior vermis Wernick/Vernick (degernation in cerebellum of alcholic patients)

****Patient has lesion of the corticospinal tract on one side in the pons. What can you expect? A. Ipsilateral spastic paresis of the body B. Contralateral spastic hemiparesis C. Contralateral los of vibration of the body D. Tongue deviate to the side of lesion and contralateral body hemiparesis E. Ataxic gate

B C - lesion in the spinal cord. D - lesion in media amaster in teh brain stem E - in the mediamedullar F. lateral lesion Usually thrombosis or embolism of the basilar artery or its branches paramedian are corticospinal tracts, the pontine nuclei, and the fibers passing to the cerebellum A laterally situated infarct will involve the trigeminal nerve

*A man was shot in a hunting accident. In the ED it was determined that the bullet hit the ALS at the level of T2 on the right side. This lesion would cause: A. loss of pain and temp at the level of C8 on the left B. loss of pain and temp at the level of T4 on the left C. loss of pain and temp at the level of C8 on the right D. loss of tactile sensation at the level ofT2 on the right E. loss of tactile sensation at the level ofT2 on the left

B Lesion in T4 Pain in T2 You go up two from lesion

Neighbors invite police due to unpleasant odor spreading from one apartment in the building. Police find in the apartment 60 year old woman unconsciousness with empty bottles of alcohol around her. She was brought to the hospital and was started on intravenous fluids with dextrose and alcohol detoxication. Over the next few days, she recovers, but ,neurologic examination shows substantial difficulty with short-term memory, nystagmus, ophthalmoplegia and ataxia. Which of the following identifiers from the brain image indicates the neural structure most likely to be damaged in this patient? A.A B.B C.C D.D E. E F.F

B (hypothalaus) Vitamine B1 deficiency Veniki Syndromes A. fornix from hypothalamus C. Pons D. Stalamus E .Splinter Macorpus Collusm (Mid brain) FF = corpus callosum Creqkyvenik Disease

Conus medullary or kauda equine Which is true regarding this structure pointed by arrow a.It is attached to the conus medullaris. b. It is a continuation of the pia and ependyma of the spinal cord. c. It penetrates the dura at the end of the dural sac (vertebra S2). d. It terminates as the coccygeal ligament fusing with the periosteum of the coccyx

B. Film terminal which is made up of: It is made up of the medulla

A 25-year-old woman presented to general practitioner with a 9-month history of low back pain and difficulty in walking, difficulty ascending and descending stairs, as well as getting in and out of chairs. After all findings were unremarkable, a malignancy workup including chest radiograph, mammogram, and CT of the abdomen were done and was positive for lung cancer. This patient's condition is most likely associated with which of the following? A.Segmental axonal demyelination B.Antibodies against presynaptic calcium channels C.Antibodies against postsynaptic acetyl choline receptors D.Medication-induced muscle fiber atrophy E.Perifascicular muscle inflammation

B. Paranailplastic Syndrome. Sometimes you cannot find tumors but you can find the nueron issues. Tumor release substance on body on the CNS. Mysathenia Gravis (antibodies against mylentiona. We can have it in the CNS like MS).

How will you know if it is an injury to hte brain stem or spinal cord?

Brain stem alwo crainl nerve will be affected. Spinal cod will not have a crainal never ipactees.

Eye Injury (Superior Rectus)

C

A 50 year-old man presented with the disease that started in childhood with the appearance of multiple hyperpigmented skin macules. At the age of 45 a lot of cutaneous tumors appeared and started to increase in size all over the body surface The cells composing these skin tumors most likely originated from which of the following structures? A.Surface ectoderm B.Neuroectoderm C.Neural crest D.Mesoderm E.Endoderm F. Notochord

C Nuerofibrillan

A 30 year female was brought to ED after traffic accident. She was on the co-driver seat and was wounded by dislocated driver seat. She is alert and fully oriented. There are a number of bruises on her head. Neurologic examination suggests a spinal cord injury at the level shown in the illustration below Which of the following spinal cord levels is most likely to be injured? A.C1 B.C7 C.T4 D.L5 E.S2

C - Thoracic lower spinal cord is in the Thoracic region. S2 is for the back of the calf and thigh area.

* Currently, the best strategy for preventing further damage in patients with acute spinal cord injury is: A. High dose corticosteroids B. Immediate exploratory surgery C. Maintenance of oxygenation and spinal cord perfusion D. Intravenous diuretic therapy

C is most improtant But A is also important after you fix C.

A 30-year-old soccer player presents to your office complying of becoming clamps. He also complained that he cannot make distinguish if water is could or warm when he is washing his hands. Neurologic examination reveals 1+ biceps reflexes and 3+ patellar reflexes bilaterally,as well as decreased muscle strength in the bilateral upper and lower extremities. Which of thefollowing is the most likely cause of this patient's problems? A.Amyotrophic lateral sclerosis B.Guillain-Barre syndrome C.Syringomyelia D.Normal pressure hydrocephalus E.Multiple sclerosis F.Medulloblastoma

C. Both sides are impacted. B C D. Inconstience, movement problem, and dementia. AlS only motor no sensory GB ascending neuropathy

7 What spinal cord level involves the Dorsal Spinocerebellar tract?

C8-S3 (C8-L2)

***Images (KNOW)

CN V spinal nucleus - Problems swallowing, speech, hoarse Vestibular nuclei - it is not the pon it is the

Central Cord Syndrom (Springymelia) - C5 and C6

Damage of the anterior gray and white commmissures

Fovilles's syndrome Mr Foville is 67 y/o male complaining of diplopias and weaknes in his left arm and leg. Neurological examination shows ipsilateral horizontal gaze palsy and facial nerve palsy and contralateral hemiparesis, hemisensory loss, and internuclear ophthalmoplegia. Fovilles's syndrome is caused by the blockage of the perforating branches of the basilar artery in pons. Structures affected by the infarct are the PPRF, nuclei of cranial nerves VI and VII, corticospinal tract, medial lemniscus, and the medial longitudinal fasciculus.

Damages of the basilar artery

2 y/o boy with slow motor development, unsteadiness, incoordination, nystagmus and progressive enlargement of the skull suddenly developed symptoms of increased intracranial pressure , irritability, vomiting and convulsions with abnormal breathing patterns. He also has malformations of limbs, face fingers, toes and the heart. His mother was on Warfarin during the pregnancy. Prenatal results (ultrasound, amniocentesis for fetal karyotype) are not available. What is the diagnosis

Dandy-Walker syndrome he Dandy-Walker syndrome is hydrocephalus associated with a posterior fossa cyst and abnormal development of the cerebellum, a portion of the brain located near the base of the skull and important to voluntary muscle movement, balance and posture. Dandy-Walker malformation (DWM) is a brain malformation that occurs during embryonic development of the cerebellum and 4th ventricl Dandy-Walker malformation may be associated with atresia of the foramen of Magendie and, possibly, the foramen of Luschka.

What is Tremors (Parkinson Syndrome)

Differentiate from Parkinson's High-frequency tremor with sustained posture (eg, outstretched arms) Occurs with intentional movement Genetic predisposition EtOH helps - patients self-medicate Drug treatment Propranolol (beta blocker) Primidone

1 What Spinocerebellar pathway conveys proprioception from the lower limbs to the cerebellum?

Dorsal SC tract Spinocerebellar

Which part of the spinal cord is present

Dorsal column. The anterior pons The dorsal column is present in the white matter.

what is the location of a doral spinocerebella first 1st order synapse

Dorsal root ganglia for all

2 What is the location of a Dorsal Spinocerebellar 1st order synapse?

Dorsal root ganglion

9 What is the location of a Cuneocerebellar 1st order synapse?

Dorsal root ganglion

what spinocereberllar pathway conveys proprioception from the lower limbs to the cerebellum

Dorsal spinocerebellar tract.

14. A male immigrants aged 43 presented at your office due to 'weakness' of the left leg, the right leg becoming involved few days later, he experienced no much pain but complained of a 'frozen feeling' affecting the left leg up to the knee. 10 years ago, he had STD that was cured in that time. You did lab work up and assess also CSF of the patient. His cerebrospinal fluid is VDRL positive. Which of the following spinal cord segments is most likely damaged in this patient? A B.B C.C D.D E. E F. F

Dosal collum is affected F

A 75-year-old,right-handed man comes to the emergency department because of acute loss of vision in his right visual field. He denies headache, slurred speech, difficulty swallowing, weakness, numbness, and difficulty walking. His other medical problems include hypertension, atrial fibrillation, and hyperlipidemia. His blood pressure is 175/105 mm Hg and pulse is 92/min and irregular. Neurologic examination shows that he is awake, alert, and oriented. His speech is fluent, and he follows complex commands. Examination shows a right homonymous hemianopsia. The rest of the neurologic examination is within normal limits. Brain MRI reveals lesion of: :A.Anterior cerebral artery B.Anterior choroidal artery C.Artery of Percheron D.Basilar artery E. Middle cerebral artery F.Posterior cerebral artery

F It is crossing If it anterior, the patient will complain of lets issur.

A 34-year-old Asian man suffered from diarrhea after intake of oily food in his childhood and it continue during his life. Physical examination reveals decreased proprioception and hyporeflexia over the lower extremities. Laboratory findings are suggestive of mild hemolytic anemia. Which of the following conditions is the most likely cause of these findings? A. Thiamine deficiency B.Vitamin D deficiency C.Vitamin A deficiency D.Vitamin C overuse E.Vitamin E deficiency F.Niacin deficiency G.Riboflavin deficiency H.Vitamin K deficiency

E The answer is E because Vitamin E deficiency can cause hemolytic anemia.

A 60-year-old male was diagnosed with intracerebral neoplasm located in the frontal lobe. Diagnosis was confirmed by CT. If the biopsy was done and it reveals neoplastic cells that stain positive for synaptophysin and negative for glial fibrillary acidic protein, these neoplastic cells most likely originate from: A.Astrocytes B.Oligodendroglia C.Ependymal cells D.Meningothelial cells E.Neurons

E An abnormal mass of tissue that forms when cells grow and divide more than they should or do not die when they should. Neoplasms may be benign (not cancer) or malignant (cancer). Neoplastic cells can come for neurons, astrocytes and oligodendroglia. GAAF Stains positive for tumors of astrocytes, oligodendrocytes and ependymal cells GAAF stains negative then not the above. Astrocytes (they have another kind of stain).

A 50 year old man had a 25-year history of regular alcohol intake, and now he gradually increased his alcohol intake. His family stated that for the last 2 years he started his mornings with his usual "eye opener," and he had not been eating enough or regularly. He is admitted to the hospital where he undergoes acute confusion. On examination it was noted horizontal nystagmus and bilateral abducens palsy. Which of the following most likely precipitated this patient's condition? A.Excessive hydration B.Diuretics C.Vitamin supplementation D.Anticoagulants E.Antibiotics F. Glucose infusion

F

How to tell if it is a lesion in the medilias lemincus?

Carry information for touch, vibration, two point discrimination, conscious proprioception Lesions: in brain stem results in loss of discrimination of touch, vibration, pressure and conscious proprioception from the contralateral side of the body

parts of cerebellum

Cerebellum and surrounding regions; sagittal view of one hemisphere. A:Midbrain. B: Pons. C: Medulla. D: Spinal cord. E: Fourth ventricle. F: Arbor vitae. G:Tonsil. H: Anterior lobe. I: Posterior lobe

3 What is the location of a Dorsal Spinocerebellar 2nd order synapse?

Clark's Nucleus (Dorsal Horn)

A 23-year-old woman is being evaluated for the development of polyhydramnios during the 15th week of her first pregnancy. Laboratory testing finds increased α-fetoprotein in her serum, and an ultrasound finds an abnormal shape to the head of the fetus with an absence of the skull. Which of the following therapies would most decrease the probability of this abnormality occurring in subsequent pregnancies for this individual? a. Completely avoid alcohol b. Decrease dietary caffeine c. Vaccinate against rubella d. Increase dietary folate e. Increase dietary vitamin A

D

Medical student was asked what she suspect if under microscopy can find neurons with shrunken nuclei, and without Nissl substance but with intensely eosinophilic cytoplasm. What was her right answer? A. Compensatory hypertrophy B.Axona l regeneration (Not in CNS) C.Compression atrophy (Not in CNS) D.Irreversible cell injury (Metabolic) E. Metabolic encephalopathy F.Normal aging

D

Where is it impacted

D = medulla C= pons A = thalamus B = corpora quadrigemina of Mid brain Film terminale is a continuation of spinal cord Filum C The pons D medulla B mid brain = corpora quadrigemina of Mid brain A thalamus (is a big structures) also have optic chaistas near it. E is cerebellum Between A and B but closer to b is the erebral Adequate/sylvia

A 45-year-old man was referred to our neurogenetics clinic because of a 6-year history of progressive impairment of gait, postural instability. During the last year, he developed dysarthria, clumsiness in upper limbs and memory complaints along difficulties to perform daily activities, which obliged him to quit his job 5 months before the time of his first consultation. This patient most likely has neuron damage in which of the following areas? A.Substantia nigra B.Dentate nucleus of the cerebellum C.Nucleus ambiguus of the medulla D.Nucleus caudatus E.Subthalamic nucleus

D hungiton disease

A 57-year-old woman presents to the ED with a severe headache of sudden onset. She describes the headache as the most severe she has ever had. She also complains of photophobia. Medical history is significant for hypertension, and cigarette smoking. Nuchal rigidity is present on examination. CT of her head is shown below. This patient's condition is most likely caused by which of thefollowing? A.Carotid artery atherosclerosis B.Cardiac embolism C.Hypoxic encephalopathy D.Hypertensive encephalopathy E.Large vessel arteritis F.Hypertensive arteriolar sclerosis G.Cerebral amyloid angiopathy H. Rupture of saccular aneurysm I. Charcot-Bouchard aneurysm

H Severe headache and Sudden Onset = Saccular Aneurysm

A 60-year-old man presented to his primary care physician with new right-sided subjective sensory abnormalities. General physical and neurological exams were normal. CT and MRI images of the brain were obtained and presented mass lesion. Biopsy is presented below. The patient most likely suffers from A.Pilocytic astrocytoma B.Glioblastoma C.Pleomorphic xanthoastrocytoma D.Oligodendroglioma E. Ependymoma F.Colloid cyst G.Medulloblastoma H. Primary CNS lymphoma I. Meningioma J.Craniopharyngioma K. Schwannoma L. Pituitary adenoma

I.

Why is substantia nigra (midbrain) important?

IT produce dopamine. part of the midbrain; a motor area; gives rise to dopamine path that deteriorates in Parkinson's disease

What is conversion? (Muscle or MLF issues)

If patient can do conversion, then muscle is not work If patient can do conversion, then muscle is working and it is MLF

Where is the lesion on the right lateral pons?

In Cranial Nerve 7th and 8th The cranial nerve 7 and 8 are both on lateral sides. Site of the lesion: The findings indicate a lesion affecting the right lateral pons with evidence of spinocerebellar involvement. This is another case of a cerebropontine angle lesion - an acoustic neuroma (or schwannoma) right-sided limb ataxia (predominantly affecting the right upper limb): spinocerebellar pathway localises the lesion to the ipsilateral lateral brainstem. - right-sided facial numbness with loss of the corneal reflex: sensory nucleus of the CN 5 localises the lesion to the ipsilateral lateral brainstem. - right-sided hemi-facial spasms: the lesion involves the pons affecting the ipsilateral CN 7. Site of the lesion: The findings indicate a lesion affecting the right lateral pons with evidence of spinocerebellar involvement. This is another case of a cerebropontine angle lesion - an acoustic neuroma (or schwannoma)

A 60-year-old female comes to the ED due to sudden left leg weakness. Examination shows intact cranial nerves and sensory function. She has decreased strength (3/5) in her left leg only. Initial CT scan without contrast showed no abnormalities. Four weeks later, repeat brain imaging showed a 8 mm, cavitary lesion in her right internal capsule. This patient's condition is most likely caused by which of the following? A. Carotid artery atherosclerosis B. Cardiac embolism C. Hypoxic encephalopathy D. Hypertensive encephalopathys E. Large vessel arteritis F. Hypertensive arteriolar sclerosis G. Cerebral amyloid angiopathy H. Arteriovenous malformation I. Saccular aneurysm J. Charcot-Bouchard aneurysm

F F can cuase Charcot-Bouchard aneurysm Hypertensive arteriolosclerosis refers to the progressive increase in the elastic and muscular components of the wall of the arteriole induced by hypertension. With long-standing hypertension, elastic tissue forms multiple concentric layers in the intima of the arteriole.

Basal Ganglia Damage What happens in the case of a lesion that involves the contralateral STN and its afferent and efferent pathways ? (HEMI)BALLISMUS

Flyign movements of hands and body like batman in an aggressive way.. Ballisms are sudden, explosive, large-amplitude, flinging motions of the limbs. If confined to one limb or one side of the body, they are termed "hemiballisms." This motor intrusion may cause postural instability and lead to falls and cause problems with everyday activities (such as eating, cooking, or dressing), but frequently also is a significant social barrier. Ballisms are sudden, explosive, large-amplitude, flinging motions of the limbs. If confined to one limb or one side of the body, they are termed "hemiballisms." This motor intrusion may cause postural instability and lead to falls and cause problems with everyday activities (such as eating, cooking, or dressing), but frequently also is a significant social barrier

A 24-year-old man was drinking heavily with some friends on the Fourth of July weekend and fell from a second-story balcony. He struck his back on a hard object on the way down and landed in a seated position. He immediately noticed complete loss of movement and sensation in his legs. In the emergency room, exam was notable for flaccid tone and 0/5 strength throughout the lower extremities, decreased rectal tone, absent bulbocavernosus reflex, and a bilateral T10 sensory level to pinprick, touch, vibration, and joint position sense LOCALIZATION AND DIFFERENTIAL DIAGNOSIS 1. On the basis of the symptoms and signs shown above, where is the lesion? 2. What is the most likely diagnosis?

Follow the sensation around T10. (Abdominal) We have both legs are not moving. Lesion can be in both legs are in spinal cord. Spinal shock

What is Brain stem death

For consciousness, you need a cortex. is a clinical syndrome defined by the absence of reflexes with pathways through the brain stem Medical criteria A brain-dead individual has no clinical evidence of brain function upon physical examination. This includes no response to pain and no cranial nerve reflexes. Reflexes include a pupillary response (fixed pupils), oculocephalic reflex, corneal reflex, caloric reflex test, and no spontaneous respirations.

4 What landmark structure is passed through on the Dorsal Spinocerebellar Tract's to the cerebellum?

Inferior Cerebellar Peduncle (Medulla)

what landmark structure is passed through on the dorsal spinocerebellar tracts to the cerebellum

Inferior penduculm To innervate the cerebellar cortex, the nerves of the spinocerebellar tract pass through a white matter structure called the inferior cerebellar peduncle

What is Polio

Polio is a viral disease that can affect nerves and can lead to partial or full paralysis. The medical name for polio is poliomyelitis.

55 y/o pt presents with the loss of proprioception, stereognosis and graphesthesia (close eyes and draw number in hand). Motor function, sense of pain, and sensitivity to crude touch are intact. Mild cognitive impairment, including difficulty concentrating and sluggish responses He could have : tumor, trauma, B12 deficiency, atherosclerosis, tabes dorsalis... WHERE IS THE LESION?

Posterior column hyperintensity on sagittal (a) and axial (b) sections

10-year-old patient presents with headache, double vision, weakness, unsteady gait, difficulty in swallowing, dysarthria, headache, vertigo, drowsiness, nausea, and vomiting. His parents reported behavioral changes, changes in handwriting and speech. Headache is insidious and intermittent, most severe in the morning or after a nap. Vomiting is projectile, occurs usually in the morning and sometimes relieves headache. He has neck pain, stiffness, and head tilt.

Posterior fossa

What is Psudeo-Argyll ROberston

Pseudo-Argyll Robertson pupils (both do not react to light or accommodation) But wider pupiles .Lesion in central nervouse sysrem in the mid-brina.

33 y/o male presents holding his left hand and wrist with his right, complaining of decreased or absent sensation on the radial and dorsal side of his hand and wrist, and of inability to extend his wrist, thumb and finger joints. With the hand supinated (palm up) and the extensors aided by gravity, hand function may appear normal, but when the hand is pronated (palm down) the wrist and hand will drop.

RADIAL NERVE

You are preparing 65 year/old patient with long standing Rheumatoid arthritis for heart operation and you think that you performed all necessary investigations. Surgeon however refuses to operate. because you forgot to make X-ray of C spine. Why?

Rhumentoids Arthrisits Syndrome of occipitoatlantoaxial hypermobility: in patients with joints hypermobility and connective tissue weakness (Ehlers- Danlos, Marfan) or Craniocervical junction in a patient with atlantoaxial impaction. The cervicomedullary angle (CMA) .

5 Where is the location of the Dorsal Spinocerebellar Tract's termination?

Rostral/caudal vermis (Cerebellum)

What are the Vermis, Regions, and Nerve REgions

Lesion in cerebellum is always on same side. Vermal Lesion is an alcholics.

Marie-Fox lateral pontine syndrome (AICA)

Lesion in the lateral pons, including the middle cerebellar peduncle.*loss of pain and temperature of the contralateral arm and leg (spinothalamic tract) *ataxia of the ipsilateral arm and leg (spinocerebellar tract) *horner's syndrome of the ipsilateral eye (sympathetic pathway) *loss of pain and temperature of the ipsilateral face (sensory nucleus of trigeminal nerve).

What are Tumors of the Pons

Look to see if the lesion is in the middle or the side? The symptoms and signs ipsilateral cranial nerve paralysis: *weakness of the facial muscles on the same side (facial nerve nucleus) *weakness of the lateral rectus muscle *nystagmus (vestibular nucleus), *impairment of hearing (cochlear nuclei) contralateral hemiparesis

A case of deafness A 66 y/o patient is presented with right-sided deafness, that was preceded by tinnitus. Clinical examination findings are as follows: right-sided limb ataxia (predominantly affecting the right upper limb). right-sided facial numbness with loss of the corneal reflex. right-sided hemi-facial spasms.

Lost of cranial reflex. Pon issue in the 5th cranial nerve Rights side facial spams (7th cranial never affects) - the lateral side of the pons impacted (numbness, facial spasms).

Most car accidens injures:

Most cervical spine fractures: - C6 or C7, Most fatal cervical spine injuries: -craniocervical junction C1 or C2

What is Ondines Curse / central hypoventilation syndrome (CCHS

Nerve impulses arising in medulla and rhythmically: - stimulate the intercostal muscles and diaphragm — making breathing possible - regulate heartbeat - regulate the diameter of arterioles thus adjusting blood flow. -respiratory arrest The neurons controlling breathing have mu (µ) receptors, the receptors to which opiates, like heroin, bind. This accounts of or the suppressive effect of opiates on breathing. Ondine's Curse or congenital central hypoventilation syndrome (CCHS) or primary alveolar hypoventilation - respiratory arrest during sleep. Ondine's curse — or, more officially, central hypoventilation syndrome — is an often fatal respiratory disorder that occurs during sleep. Typically the disorder is congenital, but in some cases can be developed later in life, especially when the patient has also had a severe injury to the brain or brain stem.

What is not in the Midbrain at the Level of the Superior Colliculi

No penduculum

What are olives

Olives in the medulla is olives. They are for hearing.

45 y/o woman presents with several months of CANDES : 1.Cape distribution of sensory loss; 2.Atrophy of the intrinsic muscles of the hands; 3.Long tract signs (both motor and sensory, which progress to the ipsilateral leg, then the contralateral leg), 4.Neck and sub occipital pain exacerbated by neck flexion or Valsalva maneuvers; 5.Dysesthesia of the hands (numbness, tingling, and cold sensation); 6.Eleventh cranial nerve palsy; 7.Stereo anesthesia. What is the issues

Tumor compress the Medulla where the forman magnum is locaetd. Formen Magnum Syndrome' It can be also associated with Multiple Sceloris when it first begins.

7-year-old boy had a normal development up to 4 years. At this age he become clumsy and could not follow his friends in running. His uncle was bedridden for a year. This patient is most likely to experience which of the following? A.Cerebral cortex atrophy B.Atrophy of caudate nucleus C.Loss of neurons of the substantia nigra D.Degeneration of the spinocerebellar tracts E.Loss of neurons in the anterior horn of the spinal cord

d a dementia b.

43. A 30 year old patient suffer from memory loss and HIV. Imaging of the brain is presented below. patient's condition can be best described as which of the following? A.Communicating hydrocephalus B.Non-communicating hydrocephalus C.Hydrocephalus ex vacuo D.Pseudotumor cerebri E.Transtentorial herniation F.Lissencephaly

d due to tissue is deteriorated .

****What is Corticobulbar (Corticonuclear) Innervation of Cranial Nerve Nuclei

Upper motor neuro for the cranial nerves •The corticobulbar innervation of cranial nerve lower motoneurons is predominantly bilateral, in that each lower motoneuron in a cranial nerve nucleus receives input from corticobulbar axons arising from both the right and the left cerebral cortex. •The major exception is that only some of the LMNs of the facial nerve ( CN VII) receive a bilateral innervation.

A 3-day-old infant born small for a date by vaginal delivery develops a decreased level of consciousness and hypotonia. Infant was still in the neonatal intensive care unit and cranial ultrasound was immediately done. It reveals blood in the lateral ventricles. Which of the following structures is the most likely source of the bleeding? A.Bridging corticalveins B.Germinal matrix C.Meningeal arteries D.Sagittal sinus E.Vessels of the circle of Willis

b E. sub germinal matrix is in thelateral ventricle

ALS (Lou Gehrig's Disease)

degeneration motor neurons of pyramidal tracts with resulting wasting (atrophy) of skeletal muscles

What caused Vomitting

Vomiting The chemoreceptor trigger zone in medulla receives inputs from blood-borne drugs or hormones, and communicates with the vomiting center, to initiate vomiting. The CTZ is close to the area postrema on the floor of the fourth ventricle and is outside of the blood-brain barrier. The neurotransmitters implicated in the control of nausea and vomiting include acetylcholine, dopamine, histamine(H-1 receptor), substance P (NK-1 receptor), and serotonin (5-HT3 receptor). There are also opioid receptors present, which may be the mechanism by which opiates cause nausea and vomiting. The blood brain barrier is not as developed here, and drugs such as dopamine which can normally not enter the CNS may still stimulate the CTZ. Dopamine cannot cross due to tight junction.

where is the loction of the dorsal spinocerbellar tract's termination?

Where does the dorsal spinocerebellar tract terminate? cerebellum It terminates bilaterally in the anterior lobe of the cerebellum (lower cerebellar peduncle) after travelling ipsilaterally from its origin in the cervical portion of the spinal cord

A 60-year-old male presents with left arm weakness. On examination motor strength was 3/5 with increased muscle tone in the left arm and "clasp knife spasticity". Which letter presents on the picture location of the lesion in this patient? A.A B.B C.C D.D E. E

Wilson Disease B (corpal interior) A - cortonuclues - if damage, it is hungintons disease. C- isala - part of lymphic system. D and E - putona clasp knife spasticity when you have corticospinal tract and

18-year-old male patient was examined by a neuro surgeon 12 months after injury to the right forearm in which the median nerve was severed. At the initial op eration, shortly after the injury had occurred, debride- ment was performed, and the separated nerve ends were tagged with radiopaque sutures. Unfortunately, the wound was infected, and surgical repair of the nerve was deferred. Is it practical to consider repairing a peripheral nerve after a delay of 12 months?

Yes, It is best to do it with 3 to 4 weeks. Yes, it can be done because it is a peripheral nerve. Peripheral nerves the axon as can be repaired. After 3 to 4 weeks, you have to start with treatment the muscle and joint first then you can do the surgery of the nerve Unlike other tissues in the body, peripheral nerve regeneration is slow and usually incomplete. Less than half of patients who undergo nerve repair after .. .

Which tract has a lesion if coordination problems begin to occur? A. spinocerebellar B. corticospinal C. dorsal column D. anterolateral system.

a

38 years old male patient brought to GP due to quadriplegia, features of pseudo bulbar palsy. On examination extensor plantar bilaterally, spasticity, clonus and exaggerated refluxes were present and muscle atrophy with fasciculation were obvious. Few years later patient died and autopsy was done. It was find atrophic precentral gyrus, loss of neurons in the ambiguous cranial nerve nuclei, in the anterior roots of the spinal cord that were thin. The patient most likely suffered from which of the following? A.Amyotrophic lateral sclerosis B.Poliomyelitis C.Rabies D.Huntington disease E.Friedreich ataxia F.Vitamin B12 deficiency

a F - media lemincus and cortiospinal tract need a lot of of F.

A 60 year old man complains of weakness and tenderness in the hand, tingling in the palm and fourth and fifth fingers. Damage of which of the following nerves has most likely cause the symptoms? A.Ulnar B.Median C.Radial D.Musculocutaneous E.Axillary

a D - elbow issue E. should issues.

.A patient who has lost pain and temperature sensation from the level of the right little finger and the rest of the body below on the right side would most likely be caused by a lesion of the: A.left ALS at C6 B. left ALS at T2 C. right ALS at C6 D. right ALS at C8 E. anterior white commissure at C8

a C6 is a finger issue Pain C6 but lesion around C3/C4

A 42-year-old physician was involved in a skiing accident and suffered a spinal cord injury. He was brought by ambulance to the emergency room, where observation indicated he might have suffered damage to the lumbosacral region of the spinal cord. The attending physician performed a variety of simple tests to localize the level of the lesion precisely. The physician first checked for cutaneous sensation on the sole of the foot. If the physician had tested the patient's ability to sense a cotton swab brushed lightly against his thigh, he would be checking the integrity of which of the following ascending sensory tracts? a. lateral spinothalamic b. fasciculus gracilis c. anterior spinothalamic d. dorsal spinocerebellar e. trigemino-thalamic

b (fine touch)

43. A 3-year-old boy presents with gait instability. His examination is significant for proximal muscle weakness, and calf enlargement. Diagnostic studies are significant for elevated levels of enzyme, creatinine kinase. This patient's condition is most likely associated with which of the following? A.Mutation of a myelin protein gene B.Mutations of a muscle structural protein gene C.Mutation of a muscle ion channel protein gene D.Endomysial inflammatory infiltration E.Endoneural inflammatory infiltration

b . Duchenene Muscular Dsystrophy (enlarged calf) E. is for Multiple Scelorisis DMD: Gora Test 0 tell patient to go on the knees. They cannot stand up. They have to stand up with their chair to push up. Thier calves are big. They have fake hypertrophy of muscles. Duchenne muscular dystrophy

What is Lateral pontine syndrome - Marie-Foix syndrome

anterior inferior cerebellar artery and long circumferential branches of the basilar artery spinothalamic tracts (lateral syndrome): contralateral loss of pain and temperature sensation on the body corticospinal tract: contralateral (movement) hemiplegia/hemiparesis xxx cerebellar tracts: ipsilateral limb ataxia facial nerve (CN VII) nucleus: ipsilateral facial paralysis vestibulocochlear nerve (CN VIII) nuclei: ipsilateral hearing loss, vertigo and nystagmus

4.The crossed extensor reflex refers to A.Signal crosses through the dorsal horn to directly stimulate the withdrawal from nocioceptive stimulus by activating extensors and inhibiting flexors in ipsilateral leg. B. The nocioceptive signal synapses on the contralateral side and so the withdrawl signal must cross the midline to activate the extensors and inhibit the flexors in the correct leg. C. Signal transversing the midline and activation of extensors contralaterally to support the body on the opposite leg. D. Motor pathways cross in the intrafusal muscles while delivering withdrawal signal to extensors.

c

A 30-year-old male presented with complaints of bilateral burning pain in the neck. Pain was lasting few months and then disappeared. Then he again developed tingling sensation in both hands with restricted cervical movements. On MRI cerebellar tonsils were displaced lower. Patient mostly have: A.Single gene disorder B.Congenital malformation C.Secondary to trauma D.Secondary to neoplasia E.Autoimmune disease F.Vascular disease

b

A 42-year-old physician was involved in a skiing accident and suffered a spinal cord injury. He was brought by ambulance to the emergency room, where observation indicated he might have suffered damage to the lumbosacral region of the spinal cord. The attending physician performed a variety of simple tests to localize the level of the lesion precisely. The physician first checked for cutaneous sensation on the sole of the foot. If the physician had tested the patient's ability to sense a cotton swab brushed lightly against his thigh, he would be checking the integrity of which of the following ascending sensory tracts? a. lateral spinothalamic b. fasciculus gracilis c. anterior spinothalamic d. dorsal spinocerebellar e. trigemino-thalamic

b

A patient presents with muscle weakness. To assess his condition, you test his knee-jerk reflex by tapping his patella tendon with your hammer. Next you examine the jaw-jerk reflex by tapping his lower jaw with your finger. The pathways that mediate these two reflexes are similar in that both involve: a. dorsal root ganglia b. a 2-neuron, sensory to motor pathway c. pathways to the cortex through ventral posterior lateral nuclei d. cell bodies of sensory neurons in peripheral ganglia e. nuclei in the brainstem

b

In response to previous part (a), where is this nuclei located? A) rostral medulla B) caudal pons C) rostral pons D) caudal medulla E) midbrain

b

Patient experiences analgesia and thermal anesthesia on tooth pulp. What nuclei is affected? A) Oral part of spinal nucleus of V B) Interpolar part of spinal nucleus of V C) Caudal part of spinal nucleus of V D) trigeminal ganglion E) trigeminal nerve

b

Q Patient notes that food tastes funny and has irregular breathing. What nucleus is affected? A) Nucleus ambiguus B) Solitary nucleus C) Inferior salivatory nucleus D) Superior salivatory nucleus E) Dorsal motor nucleus of X

b

Questions; Cerebral adequate (Slyva Adequate)

b

Spinal Cord L2 - Urination Problem with lower limb issues

b

68-year-old Asian man presents to the emergency department after he fall from a standing height. The patient was unable to provide a history, and it was unclear if this was due to a language barrier or altered mental status. His BP is 185/100 mm Hg and pulse is 66/min. His CT scan of the head is shown below. Which of the following is the most likely cause of his symptoms A.Rupture of middle meningeal artery B.Rupture of cortical bridging veins C.Rupture of lateral striate arteries D.Rupture of berry aneurysm E. Rupture of posterior communicating artery

b Subdural Hemorrhage and rupture of B. Cortical bridging veins. If middle mengingal artery (epidural) C. intercerebral hemorrhage D same E. SUbarched space.

.On the examination of this patient you find: A. Contralateral loss of pain and temperature B. Ipsilateral flaccid paralysis C. Ipsilateral spastic weakness D. Ipsilateral Horner Syndrome E. Ipsilateral loss of vibratory sense

c A. is in picturs of D - opposite side because it is crossing in teh spinal cord.

Dorsal Column INjury

dorsal columns (fasciculus gracilis and cuneatus), bilaterally, causes the absence of light touch, vibration, and position sense, bilaterally,

A 52-year-old female with a history of hypertension and tobacco use presents with sudden onset of the worse headache of her life. On CT scan subarachnoid hemorrhage was found. Soon after admission her headache was associated with confusion, vomiting, neck stiffness, and left-sided weakness. The new symptoms are most likely caused by which of the following? A.Rebleeding B.Arterial thrombosis C.Vasospasm D.Embolism E.Vascular inflammation F.Hydrocephalus

c You cannot give vasoconstriction because you make the situation worse. A vasospasm is the narrowing of the arteries caused by a persistent contraction of the blood vessels, which is known as vasoconstriction. This narrowing can reduce blood flow. Vasospasms can affect any area of the body including the brain (cerebral vasospasm) and the coronary artery (coronary artery vasospasm).

A 60-year-old male suffers an upper pontine stroke that selectively damages neuronal pathways mediating auditory signals from the lower brainstem to other relay neurons in higher levels of the brainstem. Which of the following is the principal ascending auditory pathway of the brainstem affected by the stroke? a. Medial lemniscus b. Lateral lemniscus c. Trapezoid body d. Trigeminal lemniscus e. Brachium of the superior colliculus

c a. Medial lemniscus (hearing) b. Lateral lemniscus (hearing) c. Trapezoid body (cross signal of the fibers)/ascending and descending d. Trigeminal lemniscus (trigeminal nerve) e. Brachium of the superior colliculus (connect to superior to colliculus)

A 66-year-old man presents to the office with headache and onset of blindness in his left eye. He was suffering moderate high temperature and pelvic and shoulder girdle pain. His past history reveals nonintentional weight loss during last few months. This patient's condition is most likely caused by: A. Carotid artery atherosclerosis B. Cardiac embolism C. Hypoxic encephalopathy D. Hypertensive encephalopathy E. Large vessel arteritis F. Hypertensive arteriolar sclerosis G. Cerebral amyloid angiopathy H. Arteriovenous malformation I. Saccular aneurysm

e

Following a strenuous workout with his neighbourhood team, a right-handed, 52-year-old former professional basketball player awoke the next morning with paralysis of the right lower extremity. A neurological exam revealed an exaggerated stretch reflex. There was no disturbance of position sense, pain sensation or tactile discrimination. Where is the problem localized? A. Anterior (ventral) horn, right side. B. Cerebellum, right side. C. Posterior (dorsal) columns of spinal cord, right side. D. Left motor cortex, lateral (inferior) portion of motor map. E. Left motor cortex, medial (superior) portion of motor map.

e

While I was cooking a few days ago, I reached into the oven and the top of my hand touched the heating element. My hand immediately withdrew, but the contact caused a burn on the top of my finger. The next day my finger was red and swollen and when I washed my hands in slightly warm water (30°C), the area around my burned finger became painful. Which of the following molecules contributed to the redness, swelling, and pain? A.glutamate B.substance P C.Enkephalin D.norepinephrine E.acetylcholine

e It is a neuropeptide, acting as a neurotransmitter and as a neuromodulator.

A 45year-old male presents to the doctor's office with back pain after helping his friend to move in other apartment. Pain was radiating from the gluteal region to the foot on the same side. Physical examination reveals symmetric knee reflexes, but the ankle jerk reflex is slightly diminished on the right. Which of the following nerve roots is most likely affected in this patient? A.L2 B.L3 C.L4 D.L5 E.S1

e Buttock down to foot Ankle jerk is "slightly" diminished

A 27-year-old female was hit by a car while riding her bicycle to work. She was brought to the emergency room where she complained of severe neck pain. On physical examination she was noted to have spastic paralysis of the right upper and right lower extremities, loss of vibration and position sense on the right side, and loss of pain and temperature sensation on the left side. An MRI reveals an acutely herniated disk compressing the right side of the upper cervical spinal cord. What syndrome could explain all of these patient's symptoms? Where is the injury located? What is the explanation for the loss of pain and temperature on the opposite side of the injury?

Severe nexk pain. Same side of loss of motor side. Counter side, lost of pain and temperature. Lesion of spinothalamic tract is on the contralateral side of the body (lesion 2 and 3 First order neurons of the spinothalamic tract synapse in: substantia gelatinosa

Your 55 y/o pt presents with painless ulcers on his hands, loss of pain and temperature sensibility in a shawllike distribution across the shoulders and upper torso anteriorly and posteriorly., while light touch, vibration, and position senses are preserved. Symptoms are present in both hands. He recently started having dysesthetic pain in the neck and shoulders, deep and aching and severe. He also notice weakness and atrophy in his hands, recently progressing proximally. He remembers that 15 y ago he had neck trauma. At the first glance you noticed clawhands and Horner syndrome (ptosis, myosis, enophthlamos,

Syringomyelia Loss of pain and temperature sensibility. Subacute combined degeneration Vitamin B12 deficiency, pernicious anemia "Hassan Case Presentation" on Quizlet Syringomyelia is a disorder in which a fluid-filled cyst (called a syrinx) forms within the spinal cord. Over time, the syrinx can get bigger and can damage the spinal cord and compress and injure the nerve fibers that carry information to the brain and from the brain to the rest of the body. wit loss of pain and temperatures.

**Impact of Cranial Nerve 6th and 7th

The Cranial 6th in the middle, 7th is middle but lateral side and go around the 6th cranial nerve. So, they are damaged together sometimes Cranial Nerve 6th in the middle Cranial Nerve 7th is middle but lateral. Abducens (6th) - if issues in the eyes, it cannot move to the side.

A 19-year-old woman with a history of depression is found by her college roommate lying on the floor,vomiting and having seizures,with an empty bottle of aspirin beside her. The roommate states that she has not been taking her depression medications and has been overwhelmed by the upcoming final examinations. The roommate bought the bottle of aspirin earlier that morning. In the emergency center, the patient is lethargic and confused with a low-grade fever and is hyperventilating. Her urine drug screen is negative, but the arterial blood gases reveal an anion gap metabolic acidosis, probably as a result of the aspirin overdose. What effect does metabolic acidosis have on the respiratory system? What chemoreceptors are involved in the response to an acid-base disturbance? Are the lung stretch receptors fast- or slow-acting reflexes?

Stimulate respiratory system to work - hyperstimuation Chemoreceprors in the medulla and aortic arch. In the central nervous system in fourth ventricle

A neurological examination of a 75-year-old male reveals that when the *abdominal wall is stroked*, the *muscles of the abdominal wall* of the side of the body stimulated failed to contract. Other neurological tests appeared normal. The likely region affected includes a. C1-C5 spinal segments b. C6-T1 spinal segments c. T2-T7 spinal segments d. T8-T12 spinal segments e. L1-L5 spinal segments

The answer is d. In this case, there is a loss of superficial abdominal reflexes, which require that spinal segments T8-T12 are intact. The test for these reflexes is to stroke a quadrant of the abdominal wall with an object such as a wooden stick. The normal response is for the muscle of the quadrant stimulated to contract and for movement of the umbilicus in the direction of the stimulus.

What are the structures

The caudate and putamen are separated by penetrating fibers of the internal capsule remain joined in some places by cellular bridges

Patient 12 year/old with a dorsal medullary tumor. Desmophytic brain stem glioma. Post op: cardiorespiratory arrest with persistent neurologic deficits. Brain tumors are the most common solid tumors in __________may present with a triad of signs

children with a dorsal medullary tumor

A boy was brought to your attention because of pain, limited mobility of the mandible, spasticity of the masticatory muscles and sound that is produced in the joint during mandibular movement because of an underdeveloped mandible. Development of which of the following structures is likely also impaired in this patient? A.Styloid process B.Lesser horn of the hyoid C.Stapes D.Malleus E.Thyroid cartilage

d

A patient is brought to the ER following an MVA. On examination he has weakness of his left arm and leg and loss of fine touch on the left with loss of pain and temperature sensation on the right. This clinical picture is most consistent with: A. A complete cord syndrome B. A central cord syndrome C. An anterior spinal cord syndrome D. A left spinal cord hemisection syndrome E. A right spinal cord hemisection syndrome

d

A patient was diagnosed with a form of motor neuron disease that initially affected neurons situated in the dorsolateral aspect of the ventral horn at L1-L4. Which of the following arrangements best describes the deficit likely to be present? a. LMN paralysis involving the hand b. UMN paralysis of the upper limb c. LMN paralysis of the back muscles d. LMN paralysis of the leg e. UMN paralysis of the leg

d

A neurological examination of a 75-year-old male revealed that when the abdominal wall was stroked, the muscles of the abdominal wall of the side of the body stimulated failed to contract. Other neurological tests appeared normal. Which of the following is the most likely region of the injury? a. C1-C5 spinal segments b. C6-T1 spinal segments c. T2-T7 spinal segments d. T8-T12 spinal segments e. L1-L5 spinal segments

d 3 to 5 is Phrenic nerve c6 to t 1 - neck in b/c spinal is not moveable in the neck but in the spinal cord. t2 to T7 support the arm. If there is damage in the spinal cord, it will be parapalegic. Most support th T8-T12 - abdominal / navel area L1-L5 supply the lower part of the body/ legs/herniations.

A 11-year-old boy, previously healthy presents to the ED with a 2-day history of nausea, weakness, abdominal pain, and vomiting. Physical examination reveals that he is afebrile, blood pressure of 90/60 mm Hg, and a heart rate of 115 beats/min. Lab analysis is presented below: Serum sodium Serum potassium Blood urea nitrogen Serum creatinine Calcium Serum sodium 118 mEq/L Serum potassium 4.3 mEq/L Blood urea nitrogen 40 mg/dl Serum creatinine 1.3 mg/dl If his electrolyte abnormalities were corrected too quickly to the normal values he would most likely develop? A.Cardiac arrhythmias B.Osmotic demyelination syndrome C.Progressive supranuclear palsy D.Cerebral edema E.Carpopedal spasm

b It is the rapid correction of electrolytes. Locked In Syndrome is Occulsion of the basal artery and central pontine Sodium is low in the patient (135 to 145) Potassium 3.6 to 5.2 Blood rea nitrogen Seum creatinine B happens when the electrolytes are corrected too quickly. A.It has no connection with vignette of sodium C.Bulbar and Pseudo bulbar palsy (Above the medulla) and if supranuclear it is will impact the brain stem. E. contaction in hand and foot.

Cortiospinal and Spinothalamic

This patient cannot move Arterior artery Can flle things in the

A patient who has loss of pin prick sensation starting at the level of the nipple and downward on the right side would have a lesion in the spinal cord at which level: A.C7-C8 B. T2-T3 C. T4-T5 D. T6-T7 E. T9-T10

b Nipple is T4/T5 Lesion occured at T2/T3 Pain 2 up from Lesion.

A 24-year-old African American male presents to the emergency room after a car accident resulting in paralysis of both legs. After respiratory stabilization, imaging and neurological assessment take place. Paralysis of both legs is quickly appreciated, and the patient has lost bladder control. Imaging shows a narrowing disruption of the spinal cord at the T11-T12 region of the thoracic spine. A diagnosis of a spinal cord injury (SCI) is made. ◆ What areas are anatomically affected in spinal cord injury? ◆ What are the patient's treatment options? ◆ Is neural repair a likely outcome for this patient? Why or why not?

To stabilize spinal cord. Patient will not be able to sit if not. Cortirod shot. To repair the damage is not possible.

Wha tis Medial Medullary Syndrome

Tongue deviation •Sensation to the face is preserved, due to the sparing of the trigeminal nucleus • •The syndrome is said to be "alternating" because the lesion causes symptoms both contra laterally and ipsilaterally • Sensation of pain and temperature is preserved • because the spinothalamic tract is located more laterally in the brainstem •and is also not supplied by the anterior spinal artery (instead supplied by the posterior inferior cerebellar arteries and the vertebral arteries).

A patient was diagnosed with a form of motor neuron disease that initially affected neurons situated in the dorsolateral aspect of the ventral horn at L1-L4. Which of the following arrangements best describes the deficit likely to be present? a. LMN paralysis involving the hand b. UMN paralysis of the upper limb c. LMN paralysis of the back muscles d. LMN paralysis of the leg e. UMN paralysis of the leg

d L1-L4 innvervates muscles of the lower limb

Patient with lesion in C has lesion in which of the following structures? A. Fasciculus Gracilis B. Coricospinal tract C. Anterolateral system D. Lateral horn E. In the area of low motor neuron.

b The corticospinal tract is a white matter motor pathway starting at the cerebral cortex that terminates on lower motor neurons and interneurons in the spinal cord, controlling movements of the limbs and trunk. liession is across the middle - i tis a hemisection it will low motor neuron impacts. Cut from c to G (arms) F to G issue wth legs it happens in the cervical region all the time (corciospinal tract) Huntings Sydrom - upper ) T1) motor weakness

.A patient shows a loss of tactile sensation on the right starting at the lateral aspect of the thigh and continuing over the rest of the lower right limb. There is also a loss of pain and temperature on the left from the medial aspect of the leg and below on the left. This would be caused by a lesion of the spinal cord: A.on the left at T12 B. on the left at L2 C. on the right at T12 D. on the right at L2 E. on the right at L4

d Thigh (L1/L2) Leg (L2) Ankel (l4)

A 34-year-old woman, who has been immobilized with a sprained ankle for the past 4 days, develops a throbbing pain that has spread to her entire left leg. History reveals that she has been taking oral contraceptives for 15 years. Compared to localized pain, such as one might experience from a needle stick, which of the following is true of ischemic pain? a. Ischemic pain sensory fibers are classified as A delta (Ad) sensory fibers. b. Ischemic pain is produced by overstimulating somatic touch receptors. c. Ischemic pain is transmitted to the brain through the neospinothalamic tract. d. Ischemic pain receptors quickly adapt to a painful stimulus. e. Ischemi pain sensory fibers terminate within the substantia gelatinosa of the spinal cord.

e delta fibers Ischemic rest pain is typically described as a burning pain in the arch or distal foot that occurs while the patient is recumbent but is relieved when the patient returns to a position in which the feet are dependent

A 69-year-old man with the past history of hyperlipidemia, hypertension and cervical radiculopathy presented to the emergency department with onset involuntary, flying, right upper extremity movements that started several hours ago. Which of the following areas of the brain is most likely affected? A.Lentiform nucleus B.Substantia nigra C.Caudate nucleus D.Internal capsule E.Subthalamic nucleus F.Nucleus of thethalamus

e C WIlson / with Huntington Disease (Dancing movements) B. is Parkinson Disease. D. Stroke like issues (half side of body do not work) E. Strange position of the hand

A 55 y/o patient is brought to the ER from a severe MVA. The patient complains of burning and tingling in the upper left arm. A neurological exam reveals little change in other sensory or motor functions. Which of the following regions was most likely affected by the accident? A. Dorsal horn of the spinal cord. B Dorsal columns of the spinal cord. C. Ventral horn of the spinal cord. D. Ascending pathways in the lateral funiculus of the left spinal cord. E. Nerve roots associated with the spinal cord.

e There can be compression. They have provlem in numbness and tingling.

51-year-old woman suffers from an inability to walk. Her past medical history is positive to gastric problems. Neurological examination is presented with hypertonicity and inability for two point discrimination. Myelin layer of the spinal cord presents vacuolization and axonal degeneration involving the posterior columns and the lateral corticospinal tracts. Which of the following is the most likely cause of this patient's condition? A. Syringomyelia B. Tabes dorsalis C. Multiple sclerosis D. Amyotrophic lateral sclerosis E. Poliomyelitis F. Vitamin B12 deficiency G. Thiamine deficiency H. Anterior spinal artery syndrome

f

A 40 year old man was brought to ED after he slipped on the ice covered pavement and failed on the gluteal area. Passing by people helped him to reach ED because he was in terrible pain. Examination shows pain in the low back region with flexing of the back and raising of the legs. Pinprick in the perianal area does not cause rapid contraction of the anal sphincter. Which of the following nerve roots is most likely to be injured in this patient? A. T12 B.L2 C.L4 D.L5 E. S1 F.S4

f Glureal area and No contraction Different from previous question because this one has no contraction of anal spinchter

What is colliculus sydnrome

in the floor of the 4th ventricule. The 6th and 7th Cranial Nervese are affected The 7th cranial 6th in the middle, 7th is middle but go around the 6th cranial nerver. So, they are damaged together sometimes If

A 19-year-old female with high fewer, photophobia and positive Kernig's sign was treated with Acyclovir that in few days improve her status. What can you expect to find in CSF in this patient? Glucose Protein Cells A. normal normal 50% neutrophils B. low increased 80% neutrophils C. normal increased 90% lymphocytes D. low increased 15% lymphocytes E. normal increased 90% neutrophils

in viral, you hae li

A 35-year-old woman was admitted to the hospital for investigation . She had symptoms of analgesia and thermoanesthesia on the medial side of the left hand that persisted for 6 months. Three weeks prior to her admittance, she had severely burned the little finger of her left hand on a hot stove and was unaware that the burn had occurred until she smelled the burning skin. On physical examination, she was found to have considerably reduced pain and temperature sense involving the eighth cervical and first thoracic dermatomes of the left hand. However, her sense of tactile discrimination was perfectly normal in these areas. Examination of the right arm showed a similar but much less severe dissociated sensory loss involving the same areas. No further abnormal signs were discovered. Using your knowledge of neuroanatomy, state which tract or tracts were involved in this pathological process. Name this disease.

lateral and anterior spinothalamic tract

What is Cerebellar Sensory Vestibular (Ataxia)

tumors, hemorrhage, or infarcts in this area of the brain produce a variety of symptoms and signs If they cannot stand when eyes are closed, vestibular issues. Dorsal Cullum I spinal cord Mediaamaster - in brain stem.

QUestions Where the axons leaving the medualla?

the 12th cranial nerve leaves the medually between the pyramid and olive on the right and left side. Medullar part of the medulla. It has olive bodies is how you can tell it is a medual . 12 th cranial nerve leave ventral side between pyramid Which nerve is in the medulla (12th cranial never) the 9th move through the cerebellum. .

What is MLF Syndrome - problem in left MLF internuclear ophthalmoplegiah

problem in left MLF internuclear ophthalmoplegia When a person looks to the right, the left eye do not move (and vice versa) When none move, then both has an issues. Internuclear ophthalmoplegia (INO) is an ocular movement disorder that presents as inability to perform conjugate lateral gaze and ophthalmoplegia due to damage to the interneuron between two nuclei of cranial nerves (CN) VI and CN III (internuclear). This interneuron is called the medial longitudinal fasciculus (MLF).

What happens to a lesion in the Pon

tumors, hemorrhage, or infarcts in this area produce a variety of symptoms and signs Same side lesions *involvement of the corticopontocerebellar tracts will produce marked cerebellar ataxia - intention tumor *weakness of the lateral rectus muscle on one or both sides-CNVI *Nystagmus *weakness of the facial muscles CNVII *weakness of the jaw muscles- trigeminal nerve nucleus *impairment of hearing - cochlear nuclei *anesthesia to light touch with the preservation of appreciation of pain over the skin of the face (principal sensory nucleus of trigeminal nerve involved, leaving spinal nucleus and tract of trigeminal intact) *impairment of conjugate deviation of the eyeballs due to involvement of the MLF Contralateral: *contralateral sensory defects of the trunk and limbs (medial and spinal lemnisci). and * paralysis of the limbs *most common tumor of the brainstem astrocytoma in childhood * "pinpoint pupils - involvement of the ocular sympathetic fibers

What happens to the basal ganglia in cerebellum

•1. prefrontal area "wish" •2.frontal area "decide" •3.decisin is sent to: BG • Cerebellum (knows the primary position of body) • •Before movement start BG and Cerebellum fire • •4.Cerebellum sends information to thalamus (BG do the some) then to •5. cerebrum •6. to corticospinal tract •7.to muscles

What are th blood supplies of brain stem arteris. (KNOW)

•Brain stem arteries - anterior view 1. Posterior cerebral artery 2. Superior cerebellar artery 3. Pontine branches of the basilar artery 4. Anterior inferior cerebellar artery 5. Internal auditory artery 6. Vertebral artery 7. Posterior inferior cerebellar a. 8. Anterior spinal artery •9. Basilar artery •Vertebral arteries originate from subclavian arteries •Branches of vertebral artery: Anterior spinal artery (together with vertebral supply ventrolateral and ventral medulla) •Posterior inferior cerebellar artery(supply dorsolateral medulla) •Basilar artery supply ventral side of the pons •Basilar artery branches: •AICA anterior inferior cerebellar artery(inferior pons, middle peduncle, cerebellum) •Paramedian branches (four to six in number supply pontine parenchym- the medial basal pons) •SCA superior cerebellar artery supplies the superior surface of the cerebellum and parts of the midbrain • PCA posterior cerebral artery •

*****Big Four Pathways (Brain) ****

•Corticospinal tract -Descending motor •Spinothalamic tract -Ascending pain/temperature •Dorsal columns/Medial lemniscus -Ascending somatosensory and conscious proprioception •Spinocerebellar tracts -Ascending unconscious proprioception

What is Pons: Medial Pontine Syndrome

•Corticospinal tract-contralateral hemiparesis •Lesion of the cranial nerve VI and VII-facial weakness, VI palsy can not look on the affected side - Can not move one eye laterally •Can not wrinkle face on the some side Blubal Palsy - if lesion in below meudla Psuedobulbal Palsy - the lesion is above medulla.

Comparison Huntingont CHorea and Sydenham's Chorea

•HUNTINGTON'S CHOREA -By George Huntington -TRIAD of dominant inheritance, choreoathetosis, and dementia -Bilateral atrophy of the head of the caudate nucleus and putamen -Widespread loss of cholinergic and GABAergic neurons •SYDENHAM'S CHOREA -Immunologic disorder •Associated w/ Rheumatic fever -Protein structure of streptococcal antigen similar to that of proteins in the membrane of striatal neurons -Transient, full recovery

Rule of 4

•In the rule of 4 there are 4 rules •There are 4 structures in the midline beginning with M •There are 4 structures to the side beginning with • S •There are 4 cranial nerves in the medulla • 4 in the pons and •4 above the pons (2 in the midbrain) • •The 4 motor nuclei that are in the midline are those that divide equally into 12 except for 1 and 2, that is 3, 4, 6 and 12 •(5, 7, 9 and 10 are in the lateral brainstem)

What is locked-in syndrome?

•Locked in Syndrome (infarction of the base of the pons) • of the Basilar artery •Both corticospinal tracts involved-tetraplegia •Horizontal gaze is not possible •(lesion of both n. abducentis) •Unable to speak and swallow •(lesion of corticobulbar tract) •Possible causes of locked-in syndrome include: •Traumatic brain injury •Diseases of the circulatory system •Medication overdose •Damage to nerve cells, particularly destruction of the myelin sheath caused by) •A stroke or brain hemorrhage, usually of the basilar artery

What is Dorsal Mesencephalic Lesions

•Parinaud's syndrome, characterized by loss of up-ward gaze • large and irregular pupils, eyelid retraction, convergence, nystagmus and loss of accommodation. •2. Top of the basilar syndrome, characterized by pupillary •and visual disorders, vertical gaze palsy, delirium and •hallucinations, sensory deficits, and motor deficits. This •syndrome can result from giant basilar artery tip aneurysms, vasculitis, or as a complication of cerebral angiography

What is WEBER'S SYNDROME

•Problems with weakned and eye looks down and ou tand the mouth will move controlater becuas it is upper motoer neuron for 7th cranila nerve. •occlusion of a branch of the posterior cerebral artery •oculomotor nerve and the crus cerebri. • ipsilateral ophthalmoplegia and • • contralateral paralysis of the lower part of the face, • the tongue, and the arm and leg • The eyeball is deviated laterally because of the paralysis of the medial rectus muscle Corticospinal-contralateral hemiparesis Corticobulbar-pseudobulbar palsy: UMN CN motor weakens Exacerbated gag reflex Tongue spastic-no wasting Spastic dysarthria • there is drooping (ptosis) of the upper lid • and the pupil is dilated and fixed to light and accommodation.

What is the spino-cebellum

•Spino-cerebellum •a. structures: vermis and intermediate zone of the cerebellar hemispheresb. input: most sensory modalities, especially proprioceptive endings c. output: deep cerebellar nuclei which project to brainstem motor nuclei (red nucleus and reticular motor nuclei)d. role: adjust force, direction, and rate of movement to match intended action (feedback system)-motor execution Vermis control trunk muscles and proximal limb muscles Intermedial hemisphere control distal muscles Lateral hemisphere in motor planning

What is Tectum

•Tectum -Area posterior to the ventricular space -Only prominent in the midbrain •Superior and inferior colliculi ("tectal plate")

Rule of 4

•The 4 cranial nerves above the pons are CN1-4: •Olfactory (CN1):not in midbrain. •Optic (CN2):not in midbrain. •Oculomotor (CN3):impaired adduction, supradduction and infradduction of the ipsilateral eye with or without a dilated pupil. The eye is turned out and slightly down. •Trochlear (CN4):eye unable to look down when the eye is looking in towards the nose (superior oblique). •The 3rd and 4th cranial nerves are the motor nerves in the midbrain

Rule of 4

•The 4 cranial nerves in the pons are CN5-8: •Trigeminal (CN5):ipsilateral alteration of pain, temperature and light touch on the face back as far as the anterior two-thirds of the scalp and sparing the angle of the jaw. •Abducent (CN6):ipsilateral weakness of abduction (lateral movement) of the eye (lateral rectus). •Facial (CN7):ipsilateral facial weakness. •Auditory (CN8):ipsilateral deafness. •The 6th cranial nerve is the motor nerve in the medial pons. •The 7th is a motor nerve but it also carries pathways of taste, and using the rule of 4 it does not divide equally in to 12 and thus it is not a motor nerve that is in the midline. •The vestibular portion of the 8th nerve is not included in order to keep the concept simple and to avoid confusion. Nausea and vomiting and vertigo are often more common with involvement of the vestibular connections in the lateral medulla.

What is the Cerebro-cerebellum

•a. structures: lateral cerebellar hemispheres b. input: cerebral cortex, particularly premotor area, supplementary motor area, and parietal lobe sensory areas c. output: thalamus (ventrolateral nucleus), which projects to the primary motor cerebral cortex and the brainstem red nucleus d. role: modify the motor output to muscles as the action is being planned e. motor learning (plasticity): long term modification of motor coordination based upon motor experience; possibly associated climbing fiber synapses mediated by increase Purkinje cell calcium concentration

What is BENEDIKT'S SYNDROME

•involves the medial lemniscus- contra lateral loss of proprioception •and red nucleus •producing contralateral and involuntary movements • of the limbs of the opposite side. •CN 3- Oculomotor palsy Patient can be red nucleus in which can be trembles because of red nucleus and unstable movement. The third nerve fibers are affected and the eye can move down and out

What is important about the Cranial 4th Nerve?

4th cranial nerve that lead the brain stem from dorsal side.

Questions: Burning and Sensation in Hands (Lamina I and II)

A

.Which pathways are marked by letters9

A, B, and C Cortical spinal tract. If it is damage, it can be T8 it can be D is spinalathamic tract. (pain and temperature).

Qhat are the pathways

B cortiospinal D lateral spinaothalamic C rubiospinal follows the cortipsinal. F and G is the dorsal spinocerebellar H is posterior spinocerebellar Missing letter on left is lateral cortiospinal tract. Missing letter on the right is ?

What are the lesions in the Basal Ganglia?

Brain grows up until 25. Optic nerve is a measuring point for when the brain stops growing/developing, and eye can increase until age 25. After that There is no repairment of the CNA. But those parts are damaged and no repairing. Wilson (copper accumulation in the liver and basal ganglia)

In the traffic accident 25 year old men sitting beside driver had belt but only over abdomen, without part for shoulder. He complained to the severe back pain. If you could see the damaged cells in the spinal cord you would see cell body rounding, peripheral displacement of the nuclei and dispersion of Nissl substance to the periphery of the cells. This findings most likely indicate: A.Degenerative disease B.Irreversible cell injury C.Axonal reaction D.Compression atrophy E.Normal aging

C

While crossing the street a 60-year-old male was injured by the car. He lost consciousness for a minute and after that complained of the pain in his hip that was hurt by the car. He described the accident to police officer and refuse medical examination. Less than hour later he lost consciousness and was brought to ED, where skull x-ray shows a fracture of the temporal bone. This patient's symptoms are most likely caused by blood accumulated between the: A.Aponeurosis and periosteum B.Periosteum and bone C.Bone and dura mater D.Dura mater and arachnoid E.Arachnoid and pia mater F. Pia mater and brain surface

C It is a epidural hematoma A and B are out of the scalp D is subdural E F intercephal bleeding

Following hemisection of the spinal cord at the level of approximately T3, a patient experiences loss of pain and temperature on the left side of the leg. Which of the following tracts was affected by the hemisection of the cord that could account for this deficit? a. Right fasciculus cuneatus b. Right fasciculus gracilis c. Right spinothalamic tract d. Left spinothalamic tract e. Left corticospinal tract

C It is above T1 T2 Pain and temperature will be on the opposite side. (Contalateral)

8 What spinal cord level involves the Cuneocerebellar tract?

C2-T7

The Brain of an infant who died on his first day of life is presented below. This neonate's condition was the result of: A.Deformation B.Malformation C.Disruption D.Sequence E.Agenesis

B No separation of hemisphere holoencephalon

What is Pontecerebrellary Angel Syndrome

Cranial Nerve 8 and 7 can be affected. No lesion in the tissue of the brain stem. It is out of the brain stem.

6 What Spinocerebellar pathway conveys proprioception from the upper limbs to the cerebellum?

Cuneocerebellar Tract

Clavical Fracture

D

Extremities of Right Eye (Diabetes/Hypertension)

D

Question: Locked in syndrome

D Locked in Syndtome occlusion of the basal artery and pons Locked-in syndrome is a rare neurological disorder in which there is complete paralysis of all voluntary muscles except for the ones that control the movements of the eyes (blink)

A 2-week-old baby boy was previously healthy before he presented acutely with generalized seizures. There was no evidence of a gross neurological deficit or any significant developmental delay. The head CT demonstrated impaired cerebral cortical structure with loss of the normal cortical involutions and gyrations, giving a "smooth" appearance to the cerebral cortex. A diagnosis of lissencephaly (LIS) was made. The only other notable abnormality was some mild muscle stiffness in his extremities. An error in what process in responsible for this disease. (A) Neurogenesis (B) Proliferation (C) Migration (D) Differentiation (E) Maturation

E Lissencephaly, which literally means "smooth brain," is a rare, gene-linked brain malformation characterized by the absence of normal convolutions (folds) in the cerebral cortex and an abnormally small head (microcephaly). I

Wilson Disease

E Wilson's disease is a rare inherited disorder that causes copper to accumulate in your liver, brain and other vital organs. Most people with Wilson's disease are diagnosed between the ages of 5 and 35, but it can affect younger and older people, as well.Mar 7, 2018 Symptoms: Tremor

Question: Cranial Nerve 3

Intracranial pressures Cranial Nerve 3 is in the mid-brain close to the superior cortex. Issues at night.

old patient come to ED due to left-sided weakness and numbness. His BP is 195/100 mm Hg,and HR is 78/min. Physical examination reveals diminished power on the left side (2/5) and an upgoing plantar reflex. Brain CT without contrast is obtained immediately. The image is shown below. This patient's condition is most likely caused by which of the following? A..Carotid artery atherosclerosis B.Cardiac embolism C.Hypoxic encephalopathy D.Hypertensive encephalopathy E.Large vessel arteritis F.Cerebral amyloid angiopathy G.Arteriovenous malformation H. Saccular aneurysm I. Charcot-Bouchard pseudoaneurysms

I It is ischemia Charcot-Bouchard aneurysms are minute aneurysms (microaneurysms) in the brain that occur in small penetrating blood vessels with a diameter that is less than 300 micrometers.

A 65-year-old man is brought into the emergency room with symptoms of a stroke. His vital signs are: BP is 190/100 mm Hg, and HR is 78/min, RR 20, On examination plantar reflex of the left was dorsiflexion, CT is presented below: This patient's condition is most likely caused by which of the following? A.Carotid artery atherosclerosis B.Cardiac embolism C.Hypoxic encephalopathy D.Hypertensive encephalopathy E.Large vessel arteritis F.Cerebral amyloid angiopathy G.Arteriovenous malformation H. Saccular aneurysm I. Charcot-Bouchard pseudoaneurysms

I Plantar reflex of left foot is dorsiflexion is Charcot- Bouchard Pseduoanyerysm

Tabes dorsalis

Tertiary syphilis; spinal cord and brain; endarteritis obliterans

What conveys proprioceptive information from the body to the cerebellum.

The ventral spinocerebellar tract (or anterior spinocerebellar tract) conveys proprioceptive information from the body to the cerebellum.

Question: Crainal Nerves

The lateral side of the medulla is on left. 9th and 10th cranial nerves is affected. Low motor nerve on the left side of the medulla. Cranial Nerves 11 is trapezoid

Questions: Babsinkis

Upper part of the 6th and 7th cranial nerve term-262

Mr Foville is 67 y/o male complaining of diplopia and weakness in his left arm and leg. Neurological examination shows ipsilateral horizontal gaze palsy and facial nerve palsy and contralateral hemiparesis, hemi sensory loss, and internuclear ophthalmoplegia. What is Foville Syndrome (medial pons

affecting abducens and facial nerve nuclei (ipsilateral cranial neuropathy) corticospinal tract and medial leminiscus (contralateral)

The facial nerve is destroyed as it comes from the brain stem. What nuclei is affected? A) Nucleus ambiguus B) Solitary nucleus C) Inferior salivatory nucleus D) Superior salivatory nucleus E) Ventral cochlear nucleus

b

Patient experiences decreased parotid secretions. What nuclei is affected? A) Nucleus ambiguus B) Solitary nucleus C) Inferior salivatory nucleus D) Superior salivatory nucleus E) Ventral cochlear nucleus

c

Cortiospinal tract.

controls movement of arms, legs (limb) and trunk.

Question/answer for the previous questions

d

Parkinson Syndtome (Basal Ganaglia)

"Slowed movement" is another feature of Parkinson's disease, and is particularly noticeable with walking - a motor behavior where the center of gravity shifts forward, and placing each foot forward alternately is what keeps you from falling. Slowed movements allows the center of gravity to shift too far forward, too rapidly.

What is the function of the cerebellum?

tone posture and balance coordination of movements Motor learning •The cerebellum is situated in the posterior cranial fossa and •covered superiorly by the tentorium cerebella. •It lies posterior to the fourth ventricle, and the pons and medulla oblongata • •The cerebellum is connected to the posterior aspect of the brainstem by three symmetrical bundles of nerve fibers called the superior, middle, •and inferior cerebellar peduncles.

what Pathways that crossing in the spinal cord

• •Anterior spinothalamic tract •Lateral spinothalamic tract •Anterior corticospinal tract •Ventral spinocerebellar tract These four tracts have crossing.

What is Tardive dyskinesia (Basal Ganglia Damage)

occurs as a result of long-term use of certain anti-psychotic medications, such as those used in the treatment of schizophrenia. Characterized by periodic involuntary movements (tongue & mouth movements in particular), tardive dyskinesia is addressed by halting the use of that particular drug - but the damage is done, and the presence of these intrusive movements cannot be reversed.

Spinothalamic Lateral

pain and temperature (contralateral)

. Baby is born with affected facial features, closely spaced eyes, small head size, and clefts of the lip and roof of the mouth. MRI reveals fused cerebral hemispheres. This neonate's condition was the result of A.Deformation B.Malformation C.Disruption D.Sequence E.Agenesis

. A.Deformation B.Malformation C.Disruption D.Sequence E.Agenesis

A third-year medical student attended a lecture on the effects of trauma on the vertebral column. The orthopedic surgeon described very superficially the different neurological deficits that may follow injury to the spinal cord. At the end of the lecture, the student said he did not understand what was meant by the term spinal shock. He could not understand what the underlying mechanism for this condition was. He also asked the surgeon to explain what was meant by paraplegia in extension and paraplegia in flexion. Could the surgeon explain why one condition sometimes passes into the other? These are good questions. Can you answer them?

1) temporary shock of the spinal after a injury.

Cerebral palsy: Basal Ganglia

17 y/o male with involuntary spontaneous continuous wormlike movements that are nonrhythmic, slow, writhing, sinuous...predominantly in distal muscles, often alternating with postures of the proximal limbs to produce a continuous, flowing stream of movementof a hand, an arm, the neck, face (athetosis) + nonrhythmic, jerky, rapid, nonsuppressible involuntary movements, mostly of distal muscles or the face. PMH: Prematurus with Dg Kernicterus due to Rh incompatibility. Expl: BBB is immature in newborns. Bilirubin penetrates the brain and is deposited in cell bodies (gray matter), especially the basal ganglia, causing irreversible damage. Depending on the level of exposure, the effects range from clinically unnoticeable to severe brain damage and even death.Cerebral palsy is caused by damage to the motor control centers of the developing brain and can occur during pregnancy, during childbirth or after birth up to about age three.

4.Which letter has neurons in the dorsal root ganglia 5. Which pathway has axons that cross the midline in the cord 6 Cross section is done through which area

4. Dorsal root ganglia will receive impulses from C fine touch and vibration pathways. It is anot D because it so spinal tract. A is motor 5. B 6. Lumbar area

Patient experiences analgesia and thermal anesthesia on her right side of her face and her scalp. What nuclei is affected? A) Oral part of spinal nucleus of V B) Interpolar part of spinal nucleus of V C) Caudal part of spinal nucleus of V D) trigeminal ganglion E) trigeminal nerve

5) Its B - Fasciculus gracilis fine touch 6) B - Solitary Nucleus 7) Nucleus ambiguous 8) B - Solitary nucleus 9) B - caudal pons10a) inferior salivatory nucleusso 13 is a but the professor said it is c 7 is A

Image (Test)

5th Cranial nerve is above the 7th, if the lesion is in the 5th, then it will be contralateral and impact the side of the mouth when the damage was on the

7 Patient has fewer neural crest cells than normal. Which structure. will have fewer axons? 8. Which letters are sites of degeneration in a patient with ALS

7. E (neurocrest cells will give DRG.) 8. D and C Cortical tract

lamina

A

white matter of spinal cord

A

After sitting with one leg crossed under the other for several hours while working on a document on her computer terminal, a 52 year old woman tries to stand up, but is unable to walk on the crossed leg, and feels tingling and pain. Which of the following explains the loss of motor function without the loss of pain sensation in the peripheral nerves? A. A fibers are more susceptible to local anesthetics and C fibers B. C fibers are more sensitive to pressure than AB fibers C. C fibers are more susceptible to hypoxia than B fibers D. C fibers have higher conduction velocities than A fibers E. AB fibers are more sensitive to pressure than C fibers

A B autonomic system are system to low oxygen with ischemia

A patient displays a symmetrical loss of pain and temperature on the shoulder area on both sides of the body with no loss of tactile sensation. Pain and temperature and tactile sensation are normal over the rest of the body. This condition would be due mostly likely to: A.syringomyelia B. Brown-Sequard syndrome C. vitamin B12 deficiency D. Arnold Chiari syndrome E. tabes dorsalis

A Loss of pain and temperature disordr (Syringomyelia0

. A 45-year-old man presented with a 3-day history of progressive lower limb weakness and urinary retention. Three days prior to this visit, he had fever and symptoms of upper respiratory tract infection. On examination blood pressure was 130/78 mmHg, heart rate 92 bpm, temperature 37°C,. His lower limb examination revealed a muscle power of 0/5, absent tendon reflexes at the patellas and ankles, bilateral plantar response was normal and reduced sensation to soft touch and pin prick. This patient's condition is most likely associated with which of the following? A.Endoneural inflammatory infiltration B.Toxin penetration through blood-nerve barrier C.Endoneurial arteriole hyalinization D.Mutations of a muscle structural protein gene E.Endomysial inflammatory infiltration

A MS

A 68-year-old female come to GP due to urine incontinence, unstable gate and forgetfulness. Urinary incontinence in this patient is most likely caused by which of the following? A.Stretching of descending cortical fibers B.Stretching of cerebellar inhibitory fibers C.Impaired basal ganglia signaling D.Impaired reticular formation functioning E.Spinal cord damage

A - Answer B. shaking C. problem with gait D. E. motor or sensory functions.

A 25 year old woman was sent to the PET scan examination due to involuntary dancing movements. On the examination abnormal metabolic activity in the caudate nucleus was faund. Which of the following on the gross brain section slide corresponds to the location of the abnormalities in this patient? A. A B. B C. C D. D E. E

A Huntingson Syndrom Caudate Nucleaus Nuclues Caudate (Same)

Nystagmus is an involuntary rhythmic side-to-side, up and down or circular motion of the eyes that occurs with a variety of conditions.

A commissure is side to side fibers in the brain. si

* A man fell off a ladder while working on his house and was rushed to the ED. An MRI showed damage at the level of T12 on the LEFT side in the anterolateral quadrant of his spinal cord. This injury would result in a loss of some sensation: A. from the entire leg on the right side B. from the entire leg on the left side C. from the knee down on the right side D. froms the umbilicus down on the right side E. from the umbilicus down on the left side

A. from the entire leg on the right side The spinothalamic tract (also known as anterolateral system or the ventrolateral system

A 50 year old man with uncontrolled hypertension was brought to ED after his colleagues could not understand what he is talking. Patient was frustrated and anxious due to his speech and his blood pressure rises from 165/100 to 180/100. On examination patient obey the commands and was obvious that patient understand what is told to him. Which of the are presented on the picture is affected? A.A B.B C.C D.D E.E

A.A B.B C.C D.D E.E

transverse sinus thrombosis

A woman complains of headaches after starting oral contraception. Neuroexam. reveals only ataxia. MRI shows transverse sinus thrombosis.

A 5 year-old boy was presented with unstable walking and a headache lasting for more than 10 days. ON neurological examination cerebellar ataxia and finger-to-nose test were positive, the muscle strength was normal. Preoperative brain magnetic resonance imaging (MRI) revealed a cerebellar mass. Biopsy of the tumor reveals a well-differentiated neoplasm comprised of spindle cells that have hair-like glial processes and are associated with microcysts. Based on these findings, the most likely diagnosis is which of the following? A.Pilocytic Astrocytoma B.Glioblastoma multiforme C.Medulloblastoma D.Ependymoma E.Neuroblastoma

A. Spindle cells in Schwann cells Ataxia is a term for a group of disorders that affect co-ordination, balance and speech. Any part of the body can be affected, but people with ataxia often have difficulties with: balance and walking. speaking. swallowing.

Tics and Tourette Syndrome (Basal Ganglia) Damage

A tic is a sudden, repetitive, stereotyped, nonrhythmic, involuntary movement (motor tic) or sound (phonic tic) that involves discrete groups of muscles. Tics can be invisible to the observer - such as abdominal tensing or toe crunching. Tics must be distinguished from symptoms of other disorders, such as choreas, autism, seizures, or obsessive-compulsive disorder.

Below is presented cerebral arteriography of 50 year old man. Occlusion of the artery signed by 'X' would most likely result in an inability to: A. Climb stairs B.Grip C.Whistle D.Swallow food

A. Climb stairs B.Grip C.Whistle D.Swallow food B - middle/lateral cerebral (lateral) artery C - middle/lateral cerebral (lateral) artery D - lateral side issue ( middle cerebral artery)

An 18-year-old female, who is wheel-chair bound, states that since she was a 9-year-old girl, she began to suffer from an irregular, staggering gait with frequent falls. Over the next several years, her upper extremity coordination deteriorates. She later develops weakness during her teenage years. Physical examination reveals abnormal eye movements, slow and poorly coordinated movements of all her extremities, and severe weakness of her legs. She has deficits in position sense, vibratory sense, pain, and tactile discrimination. Her deep tendon reflexes are absent. Hypertrophic cardiomyopathy was reported, and she is subsequently diagnosed with.... What is the mechanism of this disease pathology? Which pathways are involved? What are the sensory modalities lost with each pathway?

A. Spinocerebullar Tract issue (Criticalotaxia) Problem with motor and sensory B. Spinal tract is impacted then cerebellular C. Spiniocerebrullar is with sense, vibratory sense, pain, tactile discrimination and Hypertrophic cardiomyopathy Spinothalamic is with pain and temperature.

What is treatment for Parkinson Syndrome

L-dopa is used to treat Parkinson's disease. While very effective in the earlier stages of the disorder, symptoms continue to develop and larger doses of drug are required. Eventually the side effects if high doses of L-dopa will impair function and diminish quality of life. This is an example of a drug regime that allows the symptoms of a disorder to be managed - at least for a time - but is NOT a cure for the disorder. This example also shows three stages of drug action - prior to treatment (poverty of movement; resting tremor; stooped posture; shuffling gait), soon after administration (intrusive movements), and later on - during the therapeutic phase of the drug's action.

12 What spinal cord level involves the Ventral Spinocerebellar tract?

L1-S2 END of QUIZ Questions

what is the location of a dorsal spinocerebellar 2nd order synapse

L2/L3 dorsal

What is in the middle of the medulla which cranial nerve (KNOW)

(12th) Cranial- for the tongue. Where does it appears Between the cortical tract pyramid and the olive

Emma is a 64-year-old woman who has had heart disease for many years. While carrying chemicals down the stairs of the dry-cleaning shop where she works, she suddenly lost control of her right leg and arm. She fell down the stairs and was able to stand up with some assistance from a co-worker. When attempting to walk on her own, she had a very unsteady gait, with a tendency to fall to the right side. Her supervisor asked her if she was all right, and noticed that her speech was very slurred when she tried to answer. He called an ambulance to take her to the nearest hospital. Upon admission, her face appears symmetric, but when asked to protrude her tongue, it deviates toward the left. She is unable to tell if her right toe is moved up or down by the physician when she closes her eyes, and she can't feel the buzz of a tuning fork on her right arm and leg. In addition, her right arm and leg are markedly weak. The physician can find no other abnormalities in the remainder of Emma's general medical examination. Where in the nervous system could a lesion occur that would cause arm and leg weakness but spare the face? a. Right corticospinal tract in the cervical spinal cord b. Left inferior frontal lobe c. Right medullary pyramids d. Occipital lobe e. Right side of basilar pons

A Media medulla syndrome issue There is no issue on the face. It moved the lesion below the pons. There is issues in the tongue and deviated to the left and so it is lower motor neuron issue and the muscle pushed toward the lesion. Tongue to right is a upper motor neuron. C - cortiospinal trace issue on right side. D - occipatel lobe - for eyes. E. - it be upper motor neurons.

An 18-year-old man was admitted to the hospital following a severe automobile accident. After a complete neurological investigation, his family was told that he would be paralyzed from the waist downward for the rest of his life. The neurologist outlined to the medical personnel the importance of preventing complications in these cases. The common complications are the following: (a) urinary infection, (b) bedsores, (c) nutritional deficiency, (d) muscular spasms, and (e) pain. Explain the underlying reasons for these complications. How long after the accident do you think it would be possible to give an accurate prognosis in this patient?

A a) they do not feel the parasympathetic doe snot work and can leak urine and it causes infections. b) if you do move the patient, then the bone compresses the bone and the between and make ischemia. So you have to move the patient to put soft material to feel comfortable. c) need food to eat d) usually on the edge of a trauma and spams can be in extension or flexion. e) pain can return F) spinal shock nothing is working below Atrophy in the central nervous system Prognosis: usually have the patient to return to give prognosis

A 30-year-old woman comes to the physician because of multiple injuries on her hands. On neurologic examination she was found that she did not feel pinprick sensation on both hands. Damage to which of the following spinal cord areas is most likely responsible for this patient's symptoms?

A because it is Sinloamyloas when you do not feel pain. It can be in the spinal cord or medulla and it causes you to not feel pain. Congenital insensitivity to pain and anhydrosis (CIPA) is a rare hereditary disease that causes affected individuals to be unable to feel pain and unable to sweat (anhydrosis). It is also called hereditary sensory and autonomic neuropathy type IV (HSAN IV).

What are part of Midbrain: All are contents of the interpeduncular fossa EXCEPT a) Trochlear nerve b) Oculomotor nerve c) Mamillary bodies d) Tuber cinereum

A •crus cerebri •interpeduncular fossa-posterior perforated substance • •Many small blood vessels perforate the floor of the interpeduncular fossa, and this region is termed the posterior perforated substance • •The oculomotor nerve emerges on the medial side of the crus cerebri and passes forward in the lateral wall of the cavernous sinus. All are contents of the interpeduncular fossa EXCEPT a) Trochlear nerve b) Oculomotor nerve c) Mamillary bodies d) Tuber cinereum

A 70-year-old Caucasian female with unstable gait and urinary incontinence complains of memory problems. Head CT is shown on the slide below. A.Normal pressure hydrocephalus B.Alzheimer's disease C.Parkinsonism D. Pick's disease E. Major depressive disorder

A Causes urinary incontinence

What is Central pontine myelinolysis (CPM)

A neurological disease caused by severe damage of the myelin sheath of nerve cells in the brainstem, specifically the *pons*, and is characterized by acute paralysis, dysphagia, and dysarthria and other neurological symptoms.

. A 12 years old boy presented with complaints of difficulty in swallowing. History dates back to 5 years with frequent falls during walking, clumsy staggering gait, difficulty in climbing stairs, difficulty in holding objects and deterioration of hand writing. Patient's maternal uncle also had similar complaints. MRI of the brain and spine shows degeneration of the posterior columns and spinocerebellar tracts. This patient's condition is most closely mimicked by which of the following? A Lead intoxication B. Vitamin E deficiency C. Thiamine deficiency D. Prion diseases E. Poliomyelitis F. Acute intermittent porphyria

B

A 40-year-old male presented with complaints of bilateral burning type calf pain and neck pain since a few month ago. The patient also complained of restricted movement of the back, difficulty in walking and gait ataxia. MRI reveals cerebellar tonsils extending into the vertebral canal. This patient's condition is most likely: A.Single gene disorder B.Congenital malformation C.Secondary to trauma D.Secondary to neoplasia E.Autoimmune disease F.Vascular disease

B

A 58-year-old man is brought to the emergency department highly agitated due to inability to identify one's own finger on the right hand. On the examination he speaks clearly but without comprehension, on commands he shows right left confusion and he was not able to calculate a simple arithmetic task. Branch occlusion of which of the following arteries is most likely responsible for this patient's condition? A.Anterior cerebral artery B.Anterior inferior cerebellar artery C.Middle cerebral artery D.Posterior cerebral artery E.Posterior inferior cerebellar artery

B The anterior inferior cerebellar artery (AICA) is one of three pairs of arteries that supplies blood to the cerebellum. The cerebellum receives vascular supply from three main arteries that originate from the vertebrobasilar anterior system: the superior cerebellar artery (SCA), the anterior inferior cerebellar artery (AICA), and the posterior inferior cerebellar artery (PICA). Clinical Correlation: Gerstmann syndrome is a rare disorder characterized by the loss of four specific neurological functions: Inability to write (dysgraphia or agraphia), the loss of the ability to do mathematics (acalculia), the inability to identify one's own or another's fingers (finger agnosia), and inability to make the distinctio

Patient complains of fatigue, lack of appetite or abdominal pain. You find that patient has jaundice and brown ring on the cornea. Part of the brain mostly affected in this disease is pointed on the picture by arrow. Which of the following corresponds to the most affected structure? A.Globus pallidus B.Putamen C.Internal capsule D.Caudate nucleus E.Amygdala

B Wilson Disease

30. A 35 year woman rush to her doctor due to worsening headaches, confusion, and intermittent vomiting. After neurologic examination physician immediately asked for CT. On CT:enlargement of the lateral and third ventricles and a normal-sized fourth ventricle. Where is the most likely site of obstruction in this patient? A.lnterventricular foramina of Monro B.Cerebral aqueduct C.Medial foramen of Magendie D.Lateral foramina of Luschka E.Arachnoid villi

B lateral ventricle C. and D all ventricles E. All

A 5-year-old child is brought to the ER following a fall from approxi- mately 4 feet. He is now alert, moving all his extremities, and respond- ing to touch on all four extremities, but he is somewhat irritable and has a large laceration on his chin. Which of the following is true regarding evaluating the child's spine: A. Since he is moving all extremities and appears to have intact sen- sation, no further spinal evaluation needs to be performed. B. Given the child's age, spinal imaging should be performed. C. Imaging should only be performed if cervical spine tenderness can be demonstrated. D. Spinal imaging should be arranged as an outpatient.

B only up to age 9 due to him being 5 and other injuries could occur.

Brown-Sequard syndrome (BSS)

Brown-Sequard syndrome (BSS) is a rare neurological condition characterized by a lesion in the spinal cord which results in weakness or paralysis (hemiparaplegia) on one side of the body and a loss of sensation (hemianesthesia) on the opposite side.

A 35-year-old man who had been in good health noticed that his right leg was weak. As the day progressed, he found that he was dragging the leg behind him when he walked, and finally asked a friend to drive him home from work because he was unable to lift his right foot up enough to place it on the gas pedal. He also noticed that his left leg felt a little bit numb. Finally, his wife convinced him to go to the emergency room of his local hospital. In the emergency room, he had a great deal of difficulty walking. Heinformed the physician that it started slowly several days before but he had ignored the symptoms. His language function, cranial nerves, and motor and sensory examinations of his arms were within normal limits. When the physician examined his right leg, it was markedly weak, with very brisk reflexes in the knee and ankle. Vibration and position sense in the right leg were absent. Pain and temperature testing were normal in the right leg, but these sensations were absent on the left leg and abdomen to the level of his umbilicus. Reflexes in the left leg were normal, but when the physician scratched the lateral portion of the plantar surface on the bottom side of his right foot, the great toe moved up. The remainder of the patient's examination was normal. Which of the following is the primary site of the lesion? a. Lower brainstem b. Cervical spinal cord c. Thoracic spinal cord d. Lumbar spinal cord e. Peripheral nerves

C Babinski Syndrome Absent sensation and with pain and temperature issues to a leg AND umbilicus (abdomen area) - T10 (thoracic spina cord area) which cause the great toe to move up. upper abdominal region; T10-12, lower abdominal regions (thoracic spinal cord)

Why do you cough when tickled on your ear? Stimulation of the auricular branch of the CN X supplying the ear. Why and How do you cough?

Chemoreceptors Pulmonary irritant receptors (cough receptors): respiratory tract epithelium (sensitive to both mechanical and chemical stimuli). Rapidly adapting irritant receptors on the posterior wall of the trachea, pharynx, and at the main carina. Less abundant receptors in the distal airways No receptors beyond the respiratory bronchioles. Pathways: Afferent: internal laryngeal nerve, a branch of the superior laryngeal nerve (CN X), to the medulla of the brain. No clearly identifiable "cough center" in the brain. Efferent: relevant signals transmitted back from the cerebral cortex and medulla via the vagus and superior laryngeal nerves to the glottis, external intercostals, diaphragm, and other major inspiratory and expiratory muscles. Mechanism: Diaphragm (innervated by phrenic nerve) and external intercostal muscles (innervated by segmental intercostal nerves) contract, creating a negative pressure around the lung. Air rushes into the lungs in order to equalise the pressure. The glottis closes (muscles innervated by recurrent laryngeal nerve) and the vocal cords contract to shut the larynx. The abdominal muscles contract to accentuate the action of the relaxing diaphragm; simultaneously, the other expiratory muscles contract. These actions increase the pressure of air within the lungs. The vocal cords relax and the glottis opens, releasing air at over 100 mph. The bronchi and non-cartilaginous portions of the trachea collapse to form slits through which the air is forced, which clears out any irritants attached to the respitory lining. Stimulation of the auricular branch of the vagus nerve supplying the ear may also elicit a cough. This is known as Arnold's reflex. Respiratory muscle weakness, tracheostomy, or vocal cord pathology (including paralysis or anesthesia) may prevent effective clearing of the airways.

. A 30-year-old man presented with painless decrease of vision in the right eye. Right eye fundus examination revealed significant macular edema with lipid exudates. Systemic examination, including MRI of the brain, renal ultrasonography, and CT of the abdomen revealed kidney cysts. The differential diagnosis must include A.Von Recklinghausen's disease ei B.Neurofibromatosis type 2 C.Sturge-Weber syndrome D.Von Hippel-Lindau disease E. Tuberous sclerosis F.Osler-Weber-Rendu syndrome

D Von Hippel-Lindau disease (VHL) is a rare disease that causes tumors and cysts to grow in your body. They can grow in your brain and spinal cord, kidneys, pancreas, adrenal glands, and reproductive tract. The tumors are usually benign (non-cancerous). Neurofibromatosis 1 (NF1), also called von Recklinghausen's disease, is a genetic disorder characterized by the development of multiple noncancerous (benign) tumors of nerves and skin (neurofibromas) and areas of abnormal skin color (pigmentation). Sturge-Weber syndrome (SWS) is a rare vascular disorder characterized by the association of a facial birthmark called a port-wine birthmark, abnormal blood vessels in the brain, and eye abnormalities such as glaucoma. Osler-Weber-Rendu syndrome (OWR) is also known as hereditary hemorrhagic telangiectasia (HHT). It's a genetic blood vessel disorder that often leads to excessive bleeding.

Both Pathway Lesion: Wilson Disease

Disorder of copper metabolism Accumulation of copper in tissues Liver, cornea, kidneys Lentiform nucleus (putamen + globus pallidus) Increased urine copper

* A patient with elevated heart rate and blood pressure is examined by a battery of physicians and they conclude that his condition is due to a deficiency or loss of the carotid sinus reflex. Which of the following is a component of this reflex? a. Baroreceptor afferent fibers from cranial nerve XI b. Glossopharyngeal efferent fibers c. Interneurons within the nucleus ambiguus of the medulla d. Efferent fibers contained in the intermediate component of the facial nerve e. Vagal efferent fibers

E

A 45 year old woman with a history of hypertension experienced a brief "blackout". She had complained of severe headaches, nausea, dizziness and a roaring sound in the left ear during the previous day. She also had difficulty walking. The physician found: elevated blood pressure of 165/105, speech dysarthric. Nystagmus: present. There was a facial paralysis (upper and lower) on the left side which also had diminished sensation. There was impaired pain and temperature sense from the right side of the body. At rest, her eyes deviated toward the right; on command, there was paralysis of horizontal conjugate gaze to the left. Hearing was diminished on the left side. On the left, the eye drooped and the pupil was constricted.

Explanation If one side of the face it not working it is in the pons. It is a localized issue in the pons. It is an issue of the cranial nerves (in the pons) and contralateral. The cranial nerves (some are pushed down from the pon to the medulla) - the 7th is in the pon. There is one cranial nerve associated with the pons proper, the trigeminal nerve (cranial nerve V). Three other cranial nerves are located at the pontomedullary junction: the abducens nerve (cranial nerve VI), the facial nerve(cranial nerve VII), and the vestibulocochlear nerve (cranial nerve VIII). Dizziness (vertigo), nausea and nystagmus can be the result of injury of the vestibular nerve or nucleus. The roaring sound (tinnitus) with diminished hearing on one side can be the result of injury to the cochlear nerve or nucleus on the left. Paralysis of the muscles of facial expression of the upper and lower face on the left results from injury of the left facial nerve or its motor nucleus. Diminished sensation on the left face suggests injury to the nearby descending tract and nucleus of spinal V. Diminished pain and temperature on the right side of the body is due to injury of the left spinothalamic tracts. There was difficulty walking though motor strength was generally normal. This awkwardness may be attributable to injury of the left middle cerebellar peduncle. The drooping eyelid and constricted pupil suggest a Horner's syndrome due to injury of descending autonomic fibers in the lateral reticular formation. The vestibulo-cochlear nerve/nuclei, the facial nerve/nucleus, tract/nucleus of spinal V, spinothalamic tract and middle cerebellar peduncle all lie laterally in the pons within the area of supply of the anterior inferior cerebellar artery.

What is completed Transection

Fasciclus Graclis (leg) - T6 and down Fascilculus Cunneatus (Arms) - T6 and up

* A 60-year-old man walked into the neurology clinic, and the physician paid particular attention to his gait. The patient raised his feet unnecessarily high and brought them to the ground in a stamping manner. While he was waiting for the physician, it was noticed that he stood with his feet wide apart. On questioning, the patient said that he was finding it increasingly difficult to walk and was starting to use a stick, especially when he went out for walks in the dark. The physician asked the patient to stand with his toes and heels together and to close his eyes. The patient immediately started to sway, and the nurse had to steady him to prevent him from falling. On further examination, the patient was found to have loss of muscle joint sense of both legs and was unable to detect any feeling of vibration when a vibrating tuning fork was placed on the medial malleolus of either leg. No other sensory losses were noted. Using your knowledge of neuroanatomy, name the ascending pathways that are involved, by disease, in this patient. Name a disease that could be responsible for these findings.

Fasciculus GracilisFasciculus Gracilizes Fasciculus gracilis carries sensory information associated with the DCML pathway from the lower extremities and terminates and synapses at the nucleus gracilis in the caudal medulla. It is located medial relative to the fasciculus cuneatus and travels all along the spinal cord

A 63 year old woman had weakness in her left arm and speech "thick". Neurologic examination some months later showed: -Spastic paralysis of her left limbs with increased tone and exaggerated deep tendon reflexes. - -Motor examination of the face was normal, - -Upon protrusion, the tongue pointed toward her right side; the right side of her tongue was atrophic with fasciculations. - -The sensory exam indicated that pain and temperature was bilaterally normal from the body and face - but there was loss of proprioception from the left lower extremity. Where is the lesion?

Go below the lesion. Fasciculation - low motor lesion issues. The lesion is in the medial medulla When they mention the tongue, consider it. Expl: Spastic paralysis of the left arm and leg indicates injury to the corticospinal tract somewhere along its length. tongue signs places the corticospinal lesion above the cord in the medulla, on the right side The tongue usually points to the side of a lesion of the nucleus or fibers of CN XII Atrophy of its muscles confirms a lower motor neuron lesion of the fibers of CN XII on the right side. Loss of proprioception from the left lower extremity is consistent with injury to the ventral-most fibers of the medial lemniscus on the right side. Absence of other sensory findings limit the area of involvement to the distribution of the penetrating branches of the vertebral or anterior spinal artery, which include the pyramids, exiting fibers of CN XII, and the medial lemniscus.

A 48-year-old woman suffer a sudden and severe headache. She tried to drive to ED but when enter the car she could not move her head nor left nor right, she felt stiffness and pain in the neck. Her neighbor took her to ED. Her temperature was 97.88 F, blood pressure is 165/95 mm Hg and pulse was 90/min. Her pupils are symmetric and reactive to light and accommodation. CT of her head is shown below. What is the diagnosis A.Carotid artery atherosclerosis B.Cardiac embolism C.Hypoxic encephalopathy D.Hypertensive encephalopathy E.Large vessel arteritis F.Hypertensive arteriolar sclerosis G.Cerebral amyloid angiopathy H. Rupture of saccular aneurysm I. Charcot-Bouchard aneurysm

H. Circle of WIllis is the H. Severe headache and Sudden Onset = Saccular Aneurysm Saccular aneurysms are focal protrusions arising from vessel wall weaknesses at major bifurcations on the arteries along the base of the brain. These protrusions are generally spherical in shape, although asymmetric expansion of the wall is not unusual, often resulting in a multilocular appearance. Saccular aneurysms are rounded berrylike outpouchings that arise from arterial bifurcation points, most commonly in the circle of Willis (

A 20-year-old male student celebrated the passing of an examination by drinking several beers at a party. On the way home, he drove his car head-on into a bridge abutment. On examination in the emergency department, he was found to have a fracture dislocation of the ninth thoracic vertebra with signs and symptoms of severe damage to the spinal cord. On physical examination, he had an upper motor neuron paralysis of the left leg. He also had loss of muscle joint sense of the left leg. On testing of cutaneous sensibility, he had a band of cutaneous hyperesthesia extending around the abdominal wall on the left side at the level of the umbilicus. Just below this. he had a narrow band of anesthesia and analgesia. On the right side, there was total analgesia, thermoanesthesia, and partial loss of tactile sense of the skin of the abdominal wall below the level of the umbilicus and involving the whole of the right leg. Using your knowledge of neuroanatomy, state the level at which the spinal cord was damaged. Was the spinal cord completely sectioned? If not, on which side did the hemisection occur? Explain the sensory losses found on examination in this patient.

Hemisection lesion It occurred on the right side. It was a loss of function. It needs to be on the left side. Brown Syncrown Syndrome.

What is Arnold-Chiari malformation?

Herniation of the cerebellum through the foramen magnum. Generally, Type 1 involves the cerebellar tonsils (asymptomatic) - adults Type 2 cerebellar vermis and medulla, usually associated with more symptoms such as hydrocephalus, syringomyelia with loss of pain and temperature.

35 y/o male comes to your office because of low back pain. Pain started suddenly 3 days ago after he lifted a heavy piece of furniture trying to move it from left to the right corner. Intensity of the pain is 8 on the scale of 10. It is aggravated by coughing, sneezing and movements and alleviated by rest with his knees flexed. It radiates from the left lumbosacral space down to his big toe. He feels that his left foot is weak. He has no problems with urination and empting the bowel.How the problem occurred?

Herniation that is compressed on spinal cord and spinal nerves.

A 50 year old woman had a sudden onset of dizziness and vomiting. Her family noticed that her left eyelid appeared to be drooping and she was taken to see a doctor.The neurologic exam: loss of pain and temperature sensation from her right side of the body, numbness on the left side of her face. Normal proprioception and vibratory sensation bilaterally. Ataxic gait with falling toward the left side. No spasticity, no Babinski signs. When she spoke her voice had a hoarse quality and she had a diminished gag reflex. MRI: shown.

Horner Sydrome on the same side of the lesion Expl: Dizziness and vomiting suggest a lesion above the spinal cord. Spinothalamic tract involvement on the left side is indicated by the loss of pain and temperature sensation from the right side of the body. Numbness of the face on the right is consistent with injury to the nucleus and tract of spinal V on the right side (which lies near the spinothalamic tracts). Hoarseness and an abnormal gag reflex indicates that the level of the lesion is that of CN IX and X. Normal proprioception and motor strength indicates intact medial lemniscus and pyramidal tracts; the ataxic gait is probably a "cerebellar" sign attributable to injury of the inferior cerebellar peduncle (which lies near the tract of spinal V and the spinothalamics). Dizziness and vomiting may be due to involvement of the vestibular nuclei and/or CN X. Drooping of the right eyelid is part of the Horner's syndrome; at this level it is due to injury of the descending autonomics in the reticular formation. The anatomic distribution of the involved structures indicates that the injury has occurred within the area of distribution of the posterior inferior cerebellar artery. Due to infarction, this is the most common medullary syndrome.

* 35-year-old man and his wife came to see a psychiatrist because of marital problems. The wife reported that during recent months her husband had become increasingly argumentative and had also developed occasional irregular jerking movements of the head, trunk, and limbs. The husband denied having any involuntary movements. His father and several other paternal relatives had developed a similar syndrome, caused by a devastating neurodegenerative disease that destroys the basal ganglia

Huntington's Disease Basal Ganglia

How does the brain looks: Huntington Syndrome

Huntington's disease and other choreas typically are disorders in modulation of the output from the basal ganglia. Ongoing, smoothly flowing movements are typical - but the constant movement consumes calories rapidly. The movements do not occur during sleep. A different form of chorea (now called Sydenham's chorea) was once termed St. Vitus' dance due to the frenzied limb movements and contorted facial expressions. This form of chorea can appear after rheumatic fever, but also may co-occur with obsessive-compulsive disorder - which also is associated with basal ganglia dysfunction.

§Patient 1 year old §Chief complains: increased head size, vomiting, irritability §Parent noticed the increase in head size at age of 3 months and since from that the deformity gradually increased. §Past and family history no significant §On examination: conscious, average built, afebrile, pulse 110/min, respiratory and CVS system without detectable abnormalities, CNS examination within normal limits except on head examination: increased circumference of head, shiny scalp with dilated veins, fullness of anterior fontanel, sunset appearance of the eyes, head CT: posterior fossa, forth ventricle appeared normal, dilated lateral ventricles and third ventricle, cerebral pressure atrophy. §Diagnosis?

Hydrocephalus (Cerebral Aqueduct)

A 51-year-old man came to an ophthalmology appointment complaining of 8 months of progressive visual loss and headaches. He was found to have bilateral mild papilledema, with some pallor of the right optic disc, visual fields with enlarged blind spots, and concentric loss of the peripheral visual fields in both eyes (he could see only the center of the visual field with either eye;). The remainder of his neurologic exam was normal.

Hydrocephalus due to change in cerebral aqueduct. Papilledema is associated with hyrocephalus. Papilledema is a condition in which increased pressure in or around the brain (intracranial pressure) causes swelling of the part of the optic nerve inside the eye (optic disc). Symptoms of increased intracranial pressure include headache or nausea and vomiting.

Bob is a 30 year old socially inept, single male, who for several years voiced "homophobic" and "right wing" extremist views and who experienced and engaged in a compulsion to work on his car and/or wash and dust the engine late in the evening and even after midnight. However, after neighbors repeatedly complained about the noise, he was forced to cease these acts, underwent a noticeable personality change and was soon frequenting bars late at night, taking midnight drives and having sex with men he met while intoxicated.

Injury to the Basal Ganglia basal ganglia is intimately associated with the hypothalamus, amygdala, and orbital frontal lobes

A 68-year-old woman had suffered from an infectious disorder for several days. Following recovery from this disorder, she experienced problems with BP. Before viral disease she had well controlled BP, but after a viral infection BP could not be regulated. What is the reason for the variations of her BP?

It can affect the receptor in the artic tacts in the medially and sensor for regualtions. Specialized peripheral receptors, which specifically respond to changes in blood pressure, include the carotid sinus (associated with cranial nerve IX) and the aortic arch (associated with cranial nerve X). If these receptors (or the cell bodies associated with these receptors) are damaged, then one of the fundamental regulatory mechanisms for the control of blood pressure would be disrupted. The results of such a disruption would likely lead to increases and instability in blood pressure with evidence of spiking of blood pressure. Because these sensory receptors in these structures respond to increases in blood pressure, they are, in effect, stretch receptors and are consequently referred to as baroreceptors. The principal projection of the axons associated with these baroreceptors is the solitary nucleus of the medulla, which in turn, projects to autonomic nuclei such as the dorsal motor nucleus of the vagus nerve, ventrolateral medulla, and higher regions associated with autonomic functions, which include the PAG, hypothalamus, and limbic system.

What are tremors

It gets worse when you are closer to the target Resting tremaor is in Parkinson Synderom Treatment: Beta Blocker or treatment of elipsy

A 63 year old woman had weakness in her left arm and speech "thick". Neurologic examination some months later showed: - Spastic paralysis of her left limbs with increased tone and exaggerated deep tendon reflexes. - Motor examination of the face was normal, - Upon protrusion, the tongue pointed toward her right side; the right side of her tongue was atrophic with fasciculations. - The sensory exam indicated that pain and temperature was bilaterally normal from the body and face but there was loss of proprioception from the left lower extremity. Where is the lesion?

Media Medulla Syndrome Expel: Spastic paralysis of the left arm and leg indicates injury to the corticospinal tract somewhere along its length. The additional tongue signs places the corticospinal lesion above the cord, in the medulla, on the right side. The tongue usually points to the side of a lesion of the nucleus or fibers of CN XII - hypoglossal nerv. Atrophy of its muscles confirms a lower motor neuron lesion of the fibers of CN XII on the right side. Loss of proprioception from the left lower extremity is consistent with injury to the ventral-most fibers of the medial lemniscus on the right side. Absence of other sensory findings limit the area of involvement to the distribution of the penetrating branches of the vertebral or anterior spinal artery, which include the pyramids, exiting fibers of CN XII, and the medial lemniscus.

10 What is the location of a Cuneocerebellar 2nd order synapse?

Medulla (Nucleus Cuneatus) accessory

An infant presents with a history of headaches, emesis, and lethargy, that appears intermittently and is subtle that started 3 mo ago. Other symptoms: progressively worsening ataxia involving the lower extremities, with relative sparing of the trunk and upper extremities He has also signs of brain stem involvement AND Setting sun sign

Medulloblastoma is the most common malignant brain tumor in children, accounting for 10-20% of primary CNS neoplasms and approximately 40% of all posterior fossa tumors.

Why the same person has pain with the previous question.

Muscle strain on spinal cord.

What is Anterior Cord Syndrom

Pain and termperaturein spinal cord

What is Parinaud's Syndrome or Dorsal Midbrain Syndrome

Patient cannot lookup.

Where is : connects the pons and cerebellum with the forebrain anteriorly it ascends through the tentorium cerebellum

Sagittal MRI brain - 1. Internal carotid artery 2. Vertebral artery 3. Cavernous sinus 4. Carotid canal 5. anterior cerebral artery 6. Posterior cerebral artery The oculomotor nerve emerges from a groove on the medial side of the crus cerebri and passes forward in the lateral wall of the cavernous sinus # 2 gives basilar artery in the pons

Huntingon Syndrome (Choreaofrm)

Sometimes may be considered as Schizophrenia so get a MRI to see if any defects before diagnosing with a psychiatric issues. Other intrusive movements may appear similar to those of Huntington's chorea and Sydenham's chorea, but do not arise from the same causes (genetic source or bacterial infection). These movements are termed "choreaform" and they can be significant barrier to function. This individual has been asked to touch her nose with each index. Unlike true choreas, her impairment primarily is unilateral (right side). Huntington's Disease Huntington's disease is caused by a defective "Huntington Gene" on chromosome 4. This results in the degradation of the spiny GABAergic neurons in the striatum. It is an autosomal dominant disease. Symptoms of Huntington's Disease •Choreic movements (Chorea): Rapid jerky involuntary movements of the body. These normally affect the hands and face at first. They can be masked into socially acceptable movements at first but they gradually increase over time and cause great distress to patients. •Dementia develops later. •Death usually after 10-15 years.

. A large civilian aircraft was forced to abort its takeoff because three tires had burst as the plane sped along the runway. The pilot miraculously managed to halt the plane as it veered off the runway and came to an abrupt halt in a ditch. All the passengers escaped injury, but one of the stewardesses was admitted to the emergency department with suspected spinal cord injury. On questioning, the 25-year-old patient said that although she had her seat belt fastened, she was thrown violently forward on impact. She said she could not feel anything in either leg and could not move her legs. On examination, there was complete motor and sensory loss of both legs below the inguinal ligament and absence of all deep tendon reflexes of both legs. Twelve hours later, it was noted that she could move the toes and ankle of her left lower limb and she had a return of sensations to her right leg except for loss of tactile discrimination, vibratory sense. and proprioceptive sense. She had a band of complete anesthesia over her right inguinal ligament Her left leg showed a total analgesia, thermoanesthesia, and partial loss of tactile sense. Her right leg was totally paralyzed, and the muscles were spastic. There was a right-sided Babinski response, and it was possible to demonstrate right-sided ankle clonus. The right knee jerk was exaggerated. Using your knowledge of neuroanatomy, explain the symptoms and signs found in this patient. Which vertebra was damaged?

Spinal shock at first due to being paralyzed and regaining movement. Spinal shock T 12 level Loss of touch and Right leg was paralyze Upper motor lesion. Right Babiinski Lowe motor neuron on the T9 or T 10. Tactile discrimination is the ability to differentiate information through the sense of touch. ... Tactile discrimination is something that can be stronger or weaker in different people and two major conditions, chronic pain and blindness, can affect it greatly

An 18-year-old female, who is wheel-chair bound, states that since she was a 9-year-old girl, she began to suffer from an irregular, staggering gait with frequent falls. Over the next several years, her upper extremity coordination deteriorates. She later develops weakness during her teenage years. Physical examination reveals abnormal eye movements, slow and poorly coordinated movements of all her extremities, and severe weakness of her legs. She has deficits in position sense, vibratory sense, pain, and tactile discrimination. Her deep tendon reflexes are absent. Hypertrophic cardiomyopathy was reported, and she is subsequently diagnosed with.... What is the mechanism of this disease pathology? Which pathways are involved? What are the sensory modalities lost with each pathway?

Spinocerebellar Demyleniation due to ion in myochrondia It starts on the spinocerebellar pathways. By teenagers, they are bed bound. IT impacts degeneration of spinal cord. 2) damage to the cholonia it will lose proper reception and fine touch and Spinalothalamic, look for for pain and temperatur SPinalocerebellar she will be unstable, pain, sense, tactile & Hypertrophic cardiomyopathy.

A 53-year-old widower was admitted to the hospital complaining of a burning pain over his right shoulder region and the upper part of his right arm. The pain had started 3 weeks previously and, since that time, had progressively worsened. The pain was accentuated when the patient moved his neck or coughed. Two years previously, he had been treated for osteoarthritis of his vertebral column. The patient stated that he had been a football player at college, and since that time, he continued to take an active part in the game until he was 42 years old. Physical examination revealed weakness, wasting, and fasciculation of the right deltoid and biceps brachii muscles. The right biceps tendon reflex was absent. Radiological examination revealed extensive spur formation on the bodies of the fourth, fifth, and sixth cervical vertebrae. The patient demonstrated hyperesthesia and partial analgesia in the skin over the lower part of the right deltoid and down the lateral side of the arm. Using your knowledge of neuroanatomy, make the diagnosis. How is the pain produced? Why is the pain made worse by coughing?

Spondylosis Coughing and sneezing causes more pain.

A 53-year-old widower was admitted to the hospital complaining of a burning pain over his right shoulder region and the upper part of his right arm. The pain had started 3 weeks previously and, since that time, had progressively worsened. The pain was accentuated when the patient moved his neck or coughed. Two years previously, he had been treated for osteoarthritis of his vertebral column. The patient stated that he had been a football player at college, and since that time, he continued to take an active part in the game until he was 42 years old. Physical examination revealed weakness, wasting, and fasciculation of the right deltoid and biceps brachii muscles. The right biceps tendon reflex was absent. Radiological examination revealed extensive spur formation on the bodies of the fourth, fifth, and sixth cervical vertebrae. The patient demonstrated hyperesthesia and partial analgesia in the skin over the lower part of the right deltoid and down the lateral side of the arm. -What is the diagnosis -How is the pain produced? - Why is the pain made worse by coughing?

Spongiolosis - compression of the nerve. Move pain in spinal cords when coughing.

A48-year-old man complained of "numbness and stiffness" in his arms for the past 4 months. His gait has gradually deteriorated because of unsteadiness. On examination the patient appeared older than his stated age. His hair was nearly completely gray. There was slight limitation of head movement to either side, but no pain with neck extension. His tongue was red and depilated. His gait was broad based, and he was unable to walk a straight line. He was able to stand with his feet together with his eyes open, but he nearly fell when his eyes were closed. He had normal arm coordination but was ataxic on the heel-knee-shin maneuver. Deep tendon reflexes (DTRs) were 3+ in the arms, trace at the knees, and absent at the ankles. Both plantar responses were extensor. There was a 2+ jaw jerk and a positive snout reflex. There was a stocking decrease in sensation and a marked decrease in vibration and joint position sense in the toes and ankles. Cranial nerves were normal, and there were mild problems with memory and calculation. T2-weighted MRI of the brain demonstrated extensive areas of high-intensity signal in the periventricular white matter. MRI of the spine showed a hyperintense signal along the posterior column of the spinal cord. What is the most likely diagnosis? What is the next diagnostic step? What is the next step in therapy?

Subacute combined degeneration Vitamin B12, pernicious anemia Spondylosis Spondylosis is age-related change of the bones (vertebrae) andSpondylosis is age-related change of the bones (vertebrae) and discs of the spine. These changes are often called degenerative disc disease and osteoarthritis. These changes don't always cause symptoms. But they are a common cause of spine problems that can range from mild to severe. discs of the spine. These changes are often called degenerative disc disease and osteoarthritis. These changes don't always cause symptoms. But they are a common cause of spine problems that can range from mild to severe.

Why is Substantia NIgra impmortant

Substantia Nigra enhances movement Produces dopamine No dopamine = Parkinson's Syndrome SN pars reticulata - contains GABA - projects to superior colliculus - involved in saccadic eye movements SN pars compacta - contains DA -two-way street to striatum - -involved in movement Arises from the developing mesencephalon Basal ganglia output is from globus pallidum internum GABA to thalamus

What are the impacts of Substantia Nigra

Substantia nigra The substantia nigra is an important player in brain function, in particular, in eye movement, motor planning, reward seeking, learning, and addiction These neurones project to the caudate nucleus and putamen of the basal ganglia in the forebrain (nigrostriatal pathway - utilizes DOPAMINE) degeneration of the melanin-containing cells in the pars compacta of the substantia nigra Degeneration of the pars compacta of the substantia nigra is associated with Parkinson's disease Other non pigmented subdivision of the substantia nigra called the Pars reticulata is considered to be a functional homologue of the medial segment of the globus pallidus which is also part of the basal ganglia and it produces GABA.

Where is the Thalamus

The corpus striatum is situated lateral to the thalamus divided by internal capsule, into the caudate nucleus and the lentiform nucleus Laterally to the lentiform nucleus is white matter - capsule externa Laterally to it is grey matter Claustrum that separates the external capsule from Insula

Cortiospainl and Spinothalamic INjury

The corticospinal tract controls primary motor activity for the somatic motor system from the neck to the feet. It is the major spinal pathway involved in voluntary movements.

Why is it dangerous to move a patient who is suspected of having a fracture or dislocation of the vertebral column?

The damaged bone can injury the other parts of the spine cord.

CHIEF COMPLAINT A 52-year-old right-handed man was referred to a neurologist for evaluation of weakness and difficulty walking. HISTORY The patient first noticed gait difficulty 6 months prior to the appointment. He felt "off balance" and over the next 2 months developed difficulty raising his feet off the floor while seated in a chair. A few months later his leg weakness had become worse, making it difficult to walk downstairs. In addition, his arms and hands had become weak, making it difficult for him to carry out his work as a carpenter. He also noticed constant twitching of his arm and leg muscles and painful cramps in his legs. He did not complain of diplopia, dysarthria, or dysphagia. There was no history of trauma or neck pain, and no history of toxin exposure. Family history was negative. Initial evaluation by his primary care physician included an MRI of the cervical spine, which was normal. Examination: MOTOR: Increased tone in bilateral lower extremities. Nearly continuous fasciculation in all four extremities. Atrophy present in the left hand interosseous muscles and bilateral foot intrinsic muscles. Weakness present bilaterally affecting the legs more than the arms, and the right side slightly more than the left. SENSORY: Intact light touch, pinprick, vibration, and joint position sense.

The lower neurons is damaged. The lower nueron is the peripheral nerves. Increase muscle tone is upper motor neuron. The Aphasia and dysarthria are both caused by trauma to the brain, like stroke, brain injury, or a tumor. Aphasia occurs when someone has difficulty comprehending speech, while dysarthria is characterized by difficulty controlling the muscles used for speech. diplopia (double vision) Fasciculation: brief, spontaneous contraction affecting a small number of muscle fibers, often causing a flicker of movement under the skin. It can be a symptom of disease of the motor neurons.

What cranial nerves are in pons, medulla, and midbrain (above pons): (Rule of 4)

The oculomotor nerve (III) and trochlear nerve (IV) emerge from the midbrain, the trigeminal (V), abducens (VI), facial (VII) and vestibulocochlea (VIII) from the pons, the glossopharyngeal (IX), vagus (X), accessory (XI) and hypoglossal (XII) emerge from the medulla.

Question: Pontine Lesion (Double Vision/Facial Paralysis)

The patient cannot look at the left side. The occipital nerve impacted is the (Cranial Nerve 6th abducens nerve in the medulla which is the abducens nerve)

What is lateral medullary Syndrome

They are mostly in the pons. Nystagmus (8th crainial nerve impact) Horner Syndrom (always isplateral)

Alcoholic A 49-year-old man complained of difficulty walking and frequent falls for the last 2 months. His balance had progressively deteriorated to the extent that 2 friends had to carry him to the emergency department. He described his symptoms as "rubber legs." He denied weakness, paresthesias, anesthesia, pain, incontinence, urinary urgency, headache, or diplopia. Medical history was remarkable for multiple fractures due to motorcycle accidents and bilateral rotator cuff repairs. He took no medicines and had no allergies. He smoked half a pack of cigarettes per day and drank alcohol daily, ranging from 2-12 beers as well as bourbon. He was employed as a landscaper, but was now unable to work because he couldn't walk. He was single and homeless. Family history was negative for neurologic disease. On examination, he was alert and cooperative with alcohol on his breath. Vital signs were normal except for an elevated blood pressure of 150/100 mm Hg. Medical examination was normal. Mental status exam was normal. He had normal cranial nerves, with intact eye movements -- including saccades and pursuits. Strength, reflexes, and sensory exam were normal. Coordination testing revealed hesitation upon performing finger-to-nose and erratic heel-to-shin. Station was unstable and gait was wide based. He could not tandem or stand with feet together without falling. Foot tapping was slow. Speech was normal.

Vermal Thiamic

What are the vermis and cerebellar hemispheres?

Vermis is the central, constricted area. The lateral "wings" are the cerebellar hemispheres. Cerebellar hemispheres Anterior and posterior lobes

What is the BLOOD SUPPLY TO THE BRAIN STEM

Vertebral Artery •is a branch of the subclavian that ascends through the foramina of the transverse processes of the upper 6 cervical vertebrae. •It enters the posterior fossa by passing through the foramen magnum. •The vertebral arteries continue up the ventral surface of the medulla and, at the caudal border of the pons, join to form the basilar artery

What is the Vestibulo-cerebellum

Vestibulo-cerebellum a. structures: flocculo-nodular lobe (flocculus and nodulus)b. input: vestibular organsc. output: vestibular nuclei, which connect to alpha- and gamma-motor neurons of somatic muscles and eye musclesd. role: posture and muscle tone, balance, coordination of gait; eye movements

Visceral Layer and Parietal layer

Visceral layer Autonomic nerve (sympathetic & parasympathetic). Parietal layer: Somatic innervation (spinal nerves). Pathways of pain for the visceral and parietal layer are different and the quality is different as well. Visceral pain: Dull, crampy and aching. Parietal pain: Sharp, secure and persistent

A 62-year-old male presents to his general practitioner complaining of a steady tremor in his hands, which has slowly progressed for the past 6 weeks. The patient also states that walking has become increasingly difficult although he attributes this to old age. Upon physical examination, the patient has increased muscle tone with a notably hunched posture and a resting tremor. When asked to make purposeful movements, the patient is slow to initiate the movement; however, the tremor is alleviated while moving. All cranial nerves are intact and reflexive examinations normal. The patient shows no symptoms of dementia, Alzheimer, or any other cognitive disorders. An MRI is ordered which shows mild brain atrophy appropriate for his age, but otherwise unremarkable. 1) What microscopic structures are found in the neurons of patients with this disorder and what is it? 2) What are the treatment options available? 3) Which nuclei compose the basal ganglia?

You make the diagnosis of Parkinson disease. 1. Lewy bodies 2. L Dopa but NO dopamine because dopamine will give serious side effects. L Dopa because it can cross the brain blood barrier. Dopamine cannot crosst the brain blood barrier 3). Hypokinetic Rigidity Resting Tremor (in basal ganglia (Parkinson Disease) if they stop moving their hands, they will stop) In contrast, in cerebellum lesions, the tremors continue as they get closer to an object.

A 62-year-old male presents to his general practitioner complaining of a steady tremor in his hands, which has slowly progressed for the past 6 weeks. The patient also states that walking has become increasingly difficult although he attributes this to old age. Upon physical examination, the patient has increased muscle tone with a notably hunched posture and a resting tremor. When asked to make purposeful movements, the patient is slow to initiate the movement; however, the tremor is alleviated while moving. All cranial nerves are intact and reflexive examinations normal. The patient shows no symptoms of dementia, Alzheimer, or any other cognitive disorders. An MRI is ordered which shows mild brain atrophy appropriate for his age, but otherwise unremarkable. What microscopic structures are found in the neurons of patients with this disorder? ◆ What are the treatment options available? Which nuclei compose the basal ganglia?

You make the diagnosis of Parkinson disease. 1. Lewy bodies 2. L Dopa but NO dopamine because dopamine will give serious side effects. L Dopa because it can cross the brain blood barrier. Dopamine cannot crosst the brain blood barrier 3). Hypokinetic Rigidity Resting Tremor (in basal ganglia (Parkinson Disease) if they stop moving their hands, they will stop) hypokinesia or akinesia. In contrast, in cerebellum lesions, the tremors continue as they get closer to an object.

A 50 year old woman comes in complaining of hoarseness and difficulty with swallowing. She also has nasal regurgitation of liquids. What nucleus is affected? A) Nucleus ambiguus B) Solitary nucleus C) Inferior salivatory nucleus D) Superior salivatory nucleus E) Dorsal motor nucleus of X

a

A 68-year-old female come to GP due to urine incontinence, unstable gate and forgetfulness. Urinary incontinence in this patient is most likely caused by which of the following? A.Stretching of descending cortical fibers B.Stretching of cerebellar inhibitory fibers C.Impaired basal ganglia signaling D.Impaired reticular formation functioning E.Spinal cord damage

a

An 18-year-old man was admitted to the hospital following a severe automobile accident. After a complete neurological investigation, his family was told that he would be paralyzed from the waist downward for the rest of his life. The neurologist outlined to the medical personnel the importance of preventing complications in these cases. The common complications are the following: (a) urinary infection, (b) bedsores, (c) nutritional deficiency, (d) muscular spasms, and (e) pain. Explain the underlying reasons for these complications. How long after the accident do you think it would be possible to give an accurate prognosis in this patient?

a

A 62-year-old male undergoing workup for a small stroke is discovered to have a lesion of the left medial lemniscus. Physical examination of the patient would most likely reveal which of the following? A. Loss of sensation on the left face B. Loss of sensation on the right face C. Loss of proprioception on the right trunk, right arm, and right leg D. Loss of proprioception on the left trunk, left arm, and left leg E. Loss of pain and temperature on the right trunk, right arm, and right leg F. Loss of pain and temperature on the left trunk, left arm, and right leg

c medial lemincisu does not carry pain and temperature Choice (C) is the correct answer. First order neurons of the dorsal column-medial lemniscus system are located in the dorsal root ganglion and ascend ipsilaterally where they synapse in the medulla in either the gracile nucleus or cuneate nucleus. The gracile nucleus carries sensory information, including vibration, two-point discrimination, and proprioception, from the ipsilateral lower extremity. The cuneate nucleus carries sensory information from the ipsilateral upper extremity. Secondary neurons cross the midline as internal arcuate fibers and form the medial lemniscus. The secondary order neurons continue to ascend in the contralateral medial lemniscus and synapse in the ventral posterolateral (VPL) nucleus of the thalamus. Third order neurons project from the contralateral VPL nucleus to the contralateral post central gyrus, which contains the primary somatosensory cortex. A lesion of the left medial lemniscus would result in the loss of vibration, two-point discrimination, and proprioception of the contralateral or right upper extremity, right trunk, and right lower extremity. The right face is spared. The other choices are incorrect.

A 28-year-old woman with peripheral paresis of the facial nerve, balance disturbance, hearing loss for many years is presented to her family doctor. CT scan is presented below. This patient's condition is most likely associated with a mutation involving which of the following genes? A. on chromosome 5 B.NF1 on chromosome 17 C.NF2 on chromosome 22 D.p53 on chromosome 17 E. RB1 on chromosome 13 F.VHL on chromosome 3

c neurofibromatosis type 2

Questions: Mulitpel Scelorisis

c. Burning Sensation Muscle weakness Visual Problems Impairment of gait Lhermitte's sign ( bend down head and pain happens)

.A 28 year-old man was shot in the back outside a bar during an argument. He was rushed to the ED where an exam revealed that he had no pin prick sensation in his left lower extremity. Vibratory sensation was intact on his body and limbs. The most likely location of a lesion causing these symptoms is: A.T6 B. T8 C.T12 D. L2 E. L4

c. It is usually L2 but is T12 Enter in T12 so go up to next number L2. Lesion will be on the opposite side in the spinal cord. He can still fell but not on his left side.

A 5-year-old girl is brought to Pediatrician by her mother because of pubic hair growth and changes of the breasts. Physical examination reveals pubic hair growth, and impaired upward gaze. Where is the lesion A.Site A B.Site B C.Site C D.Site D E.Site E F.Site F G.Site G

is c corpora quadrigemina he corpora quadrigemina are reflex centers involving vision and hearing. I


Related study sets

Chapter 14 - Capacity and Legality

View Set

Religious Ceremonies - birth and death

View Set

Nursing Fundamentals Chapter 31.test questions

View Set

Set theory, permutations and combinations

View Set

11th Physics | Quiz: Acceleration

View Set

Microeconomics chapter 9 - Businesses and the Costs of Production

View Set

Discovering the Universe Chpt. 2

View Set

Surah Naziaat Tafseer Test (questions)

View Set